Está en la página 1de 65

Subspecialty Day

2011 Orlando October 21 22

RO-OPHTHALMOLOGY NEU

Neuro-Ophthalmology 2011: What Is Wrong With This Picture? Recognizing the Neuro-Ophthalmic Red Flags

NeuroOphthalmology 2011 What Is Wrong With This Picture? Recognizing the Neuro-Ophthalmic Red Flags
Program Directors
Andrew G Lee MD and Michael S Lee MD

In conjunction with the North American Neuro-Ophthalmology Society


Orange County Convention Center Orlando, Florida Saturday, October 22, 2011 Presented by: The American Academy of Ophthalmology

2011 Neuro-Ophthalmology Planning Group Andrew G Lee MD Program Director Michael S Lee MD Program Director Madhu R Agarwal MD Eric L Berman MD Wayne T Cornblath MD Matthew Dean Kay MD Leah Levi MBBS Alfredo A Sadun MD PhD Prem S Subramanian MD PhD Former Program Directors: 2009 Karl C Golnik MD Leah Levi MBBS 2007 Andrew G Lee MD Karl C Golnik MD

2005 2003 2001

Anthony C Arnold MD Andrew G Lee MD Barrett Katz MD MBA Anthony C Arnold MD Barrett Katz MD MBA Neil R Miller MD

Subspecialty Day Advisory Committee William F Mieler MD Associate Secretary Donald L Budenz MD MPH Daniel S Durrie MD Robert S Feder MD Leah Levi MBBS R Michael Siatkowski MD Jonathan B Rubenstein MD Secretary for Annual Meeting

Staff Melanie R Rafaty CMP, Director, Scientific Meetings Ann LEstrange, Scientific Meetings Coordinator Debra Rosencrance CMP CAE, Vice President, Meetings & Exhibits Patricia Heinicke Jr, Editor Mark Ong, Designer Gina Comaduran, Cover Design

2011 American Academy of Ophthalmology. All rights reserved. No portion may be reproduced without express written consent of the American Academy of Ophthalmology.

ii

2011 Subspecialty Day

Neuro-Ophthalmology

Dear Colleague: On behalf of the American Academy of Ophthalmology (the Academy) and the North American Neuro-Ophthalmology Society (NANOS), it is our great pleasure to welcome you to Orlando and to Neuro-Ophthalmology 2011: What Is Wrong With This Picture? Recognizing the NeuroOphthalmic Red Flags. As co-chairs of the Academys Neuro-Ophthalmology Subspecialty Day Program Planning Group, we are honored to have the opportunity to plan this years meeting. We have assembled an outstanding faculty of internationally regarded experts, and we anticipate that this will be an extraordinary educational event. Our faculty have prepared an exciting and educational program that we hope will provide you with the most up-to-date and comprehensive review of these essential topics. On behalf of the Academy and NANOS, we thank our participants for all of their efforts and for sharing their time, experience and expertise. Todays meeting will provide examples of distinctive clinical signs crucial to making the correct neuro-ophthalmologic diagnosis. Each session is composed of several case presentations followed by a panel discussion of the individual cases to highlight how these specific symptoms and signs can allow one to reach the correct diagnosis. Finally, the session moderator will provide a short summary and open the floor to questions via text messaging. In an effort to provide the most innovative and interesting Subspecialty Day Meetings in the future, we request that you assist us by completing the meeting evaluation form. We carefully review all of your comments to better understand your needs, so please candidly indicate the strengths and shortcomings of todays program. Again, we welcome you to Neuro-Ophthalmology 2011: What Is Wrong With This Picture? Recognizing the Neuro-Ophthalmic Red Flags. We hope you find it intellectually stimulating, educational and enjoyable. Sincerely,

Andrew G Lee MD Program Director

Michael S Lee MD Program Director

2011 Subspecialty Day |

Neuro-Ophthalmology

iii

Neuro-Ophthalmology 2011 Contents

Program Directors Welcome Letter ii CME Credit iv Faculty Listing v Program Schedule xi

Case Presentations 1
Section I: What Is Wrong With This Fundus? 2 Surgery by Surgeons Update 6 Section II: Section III: Section IV: What Is Wrong With This Eyelid/Pupil? 7 What Is Wrong With This Eye Movement? 10 What Is Wrong With This Test? 12

Answer and Teaching Points 15


Section I: Section II: Section III: Section IV: What Is Wrong With This Fundus? 16 What Is Wrong With This Eyelid/Pupil? 23 What Is Wrong With This Eye Movement? 30 What Is Wrong With This Test? 37 Faculty Financial Disclosure 45 Presenter Index 49

iv

2011 Subspecialty Day

Neuro-Ophthalmology

CME Credit

Academys CME Mission Statement


The purpose of the American Academy of Ophthalmologys Continuing Medical Education (CME) program is to present ophthalmologists with the highest quality lifelong learning opportunities that promote improvement and change in physician practices, performance or competence, thus enabling such physicians to maintain or improve the competence and professional performance needed to provide the best possible eye care for their patients.

Attendance Verification for CME Reporting


Before processing your requests for CME credit, the Academy must verify your attendance at Subspecialty Day and/or the Annual Meeting. In order to be verified for CME or auditing purposes, you must either: Register in advance, receive materials in the mail and turn in the Final Program and/or Subspecialty Day Syllabus exchange voucher(s) onsite; Register in advance and pick up your badge onsite if materials did not arrive before you traveled to the meeting; Register onsite; or Use your ExpoCard at the meeting.

2011 Neuro-Ophthalmology Subspecialty Day Meeting Learning Objectives


Upon completion of this activity, participants should be able to: List key clinical symptoms that should prompt consideration of additional neuro-ophthalmic conditions Define distinctive clinical signs in common neuroophthalmic conditions that should prompt consideration for additional evaluation List distinctive radiographic signs in specific neuroophthalmic conditions Describe specific pitfalls in diagnostic testing that might lead to erroneous diagnosis in neuro-ophthalmology

CME Credit Reporting


Level 2, Lobby B; Academy Resource Center, Hall A4, Booth 1359 Attendees whose attendance has been verified (see above) at the 2011 Annual Meeting can claim their CME credit online during the meeting. Registrants will receive an e-mail during the meeting with the link and instructions on how to claim credit. Onsite, you may report credits earned during Subspecialty Day and/or the Annual Meeting at the CME Credit Reporting booth. Academy Members: The CME credit reporting receipt is not a CME transcript. CME transcripts that include 2011 Annual Meeting credits entered onsite will be available to Academy members on the Academys website beginning Nov. 16, 2011. NOTE: CME credits must be reported by Jan. 18, 2012. After the 2011 Annual Meeting, credits can be claimed at www.aao.org. The Academy transcript cannot list individual course attendance. It will list only the overall credits spent in educational activities at Subspecialty Day and/or the Annual Meeting. Nonmembers: The Academy will provide nonmembers with verification of credits earned and reported for a single Academysponsored CME activity, but it does not provide CME credit transcripts. To obtain a printed record of your credits, you must report your CME credits onsite at the CME Credit Reporting booths.

2011 Neuro-Ophthalmology Subspecialty Day Meeting Target Audience


The intended audience for this program is comprehensive ophthalmologists.

2011 Neuro-Ophthalmology Subspecialty Day CME Credit


The American Academy of Ophthalmology is accredited by the Accreditation Council for Continuing Medical Education to provide continuing medical education for physicians. The American Academy of Ophthalmology designates this live activity for a maximum of 7 AMA PRA Category 1 Credits. Physicians should claim only the credit commensurate with the extent of their participation in the activity.

Scientific Integrity and Disclosure of Relevant Financial Interest


The American Academy of Ophthalmology is committed to ensuring that all continuing medical education (CME) information is based on the application of research findings and the implementation of evidence-based medicine. It seeks to promote balance, objectivity and absence of commercial bias in its content. All persons in a position to control the content of this activity must disclose any relevant financial interest. The Academy has mechanisms in place to resolve all conflicts of interest prior to an educational activity being delivered to the learners.

Proof of Attendance
The following types of attendance verification will be available during the 2011 Annual Meeting and Subspecialty Day for those who need it for reimbursement or hospital privileges, or for nonmembers who need it to report CME credit: CME credit reporting/proof-of-attendance letters Onsite Registration Form Instruction Course Verification Form Visit the Academys website for detailed CME reporting information.

2011 Subspecialty Day

Neuro-Ophthalmology

Faculty

Marie D Acierno MD
Baton Rouge, LA Professor of Ophthalmology and Residency Director of Ophthalmology Louisiana State University Health Science Center Ochsner, New Orleans

Rudrani Banik MD
New York, NY Associate Adjunct Surgeon New York Eye & Ear Infirmary Faculty Bronx-Lebanon Hospital Center

Francois-Xavier Borruat MD
Lausanne, Switzerland Mdecin-adjoint, Neuro-Ophthalmology Hpital Ophtalmique Jules-Gonin

No photo available

Beau Benjamin Bruce MD MS Madhu R Agarwal MD


Redlands, CA Physician California Orbital Consultants

Valerie Biousse MD
Atlanta, GA Professor of Ophthalmology and Neurology Emory University School of Medicine

Atlanta, GA Assistant Professor of Ophthalmology and Neurology Emory University

Dean M Cestari MD Anthony C Arnold MD


Los Angeles, CA Professor and Chief Neuro-Ophthalmology Division Jules Stein Eye Institute

Gabrielle R Bonhomme MD
Pittsburgh, PA Assistant Professor of Ophthalmology University of Pittsburgh M.D. University of Pittsburgh Medical Center

Boston, MA

vi

Faculty Listing

2011 Subspecialty Day

Neuro-Ophthalmology

Sophia Mihe Chung MD


St Louis, MO Associate Professor of Ophthalmology and Neurology Saint Louis University School of Medicine

Fiona E Costello MD
Calgary, AB, Canada Associate Professor Department of Clinical Neurosciences and Surgery Hotchkiss Brain Institute University of Calgary

Julie Falardeau MD
Portland, OR Assistant Professor of Ophthalmology Casey Eye Institute Assistant Professor of Ophthalmology Devers Eye Institute

Kimberly Cockerham MD FACS


Los Altos, CA Adjunct Associate Clinical Professor Department of Ophthalmology Stanford School of Medicine Staff Ophthalmologist Palo Alto Veterans Health System

Kathleen B Digre MD
Salt Lake City, UT Professor of Neurology and Ophthalmology Moran Eye Center University of Utah

Rod Foroozan MD
Houston, TX Assistant Professor of Ophthalmology Baylor College of Medicine

Larry P Frohman MD Wayne T Cornblath MD


Ann Arbor, MI Professor of Ophthalmology & Visual Sciences and Neurology University of Michigan

Eric Eggenberger DO
East Lansing, MI Professor of Neurology and Ophthalmology Michigan State University

Newark, NJ Professor of Ophthalmology & Visual Science & Neurology & Neurosciences University of Medicine and Dentistry, New Jersey New Jersey Medical School Executive Vice President North American Neuro-Ophthalmology Society

2011 Subspecialty Day

Neuro-Ophthalmology

Faculty Listing

vii

No photo available

Karl C Golnik MD
Terrace Park, OH Professor of Ophthalmology, Neurology, and Neurosurgery University of Cincinnati and The Cincinnati Eye Institute Professor of Ophthalmology University of Louisville

Guy V Jirawuthiworavong MD
Torrance, CA Regional Neuro-Ophthalmologist/Uveitis Consultant Southern California Kaiser Permanente Medical Group

Matthew Dean Kay MD


Pompano Beach, FL Adjunct Clinical Associate Professor Nova Southeastern University Adjunct Assistant Professor Florida International University School of Medicine

Michael C Johnson MD Lynn K Gordon MD PhD


Los Angeles, CA Professor of Ophthalmology Jules Stein Eye Institute Associate Dean, Diversity Affairs David Geffen School of Medicine at UCLA Edmonton, AB, Canada Assistant Professor University of Alberta

Lanning B Kline MD
Birmingham, AL Chairman and Professor of Ophthalmology University of Alabama at Birmingham

Randy H Kardon MD PhD


Iowa City, IA Professor of Ophthalmology and Director of Neuro-Ophthalmology University of Iowa Professor of Ophthalmology and Director Center for the Prevention and Treatement of Visual Loss Surgery and Research Division Department of Veterans Affairs

Gregory S Kosmorsky DO
Highland Heights, OH Head, Section of Neuro-Ophthalmology Cleveland Clinic

Thomas A Graul MD
Lincoln, NE

viii

Faculty Listing

2011 Subspecialty Day

Neuro-Ophthalmology

No photo available

Byron L Lam MD
Miami, FL Professor of Ophthalmology Bascom Palmer Eye Institute University of Miami

Reid A Longmuir MD
Iowa City, IA Assistant Professor of Ophthalmology and Visual Sciences The University of Iowa Attending Physician Veterans Admistration Hospital

Mark L Moster MD
Merion Station, PA Neuro-Ophthalmology Service Wills Eye Institute Professor of Neurology Thomas Jefferson University School of Medicine

Andrew G Lee MD
Houston, TX Professor of Ophthalmology, Neurology, and Neurosurgery Weill Cornell Medical College Chair, Department of Ophthalmology The Methodist Hospital

Timothy J McCulley MD
Baltimore, MD Associate Professor Johns Hopkins School of Medicine Director of Oculoplastic Surgery King Khaled Eye Spcialist Hospital, Riyadh, Saudi Arabia

Nancy J Newman MD
Atlanta, GA LeoDelle Jolley Professor of Ophthalmology and Professor of Ophthalmology and Neurology Emory University School of Medicine

Michael S Lee MD
Minneapolis, MN Professor Departments of Ophthalmology, Neurology and Neurosurgery University of Minnesota

Neil R Miller MD
Baltimore, MD Frank B Walsh Professor of NeuroOphthalmology Wilmer Eye Institute Johns Hopkins University School of Medicine Professor of Ophthalmology, Neurology and Neurosurgery Johns Hopkins University School of Medicine

Steven A Newman MD
Charlottesville, VA Professor of Ophthalmology University of Virginia Health System

2011 Subspecialty Day

Neuro-Ophthalmology

Faculty Listing

ix

Jeffrey G Odel MD
New York, NY Associate Clinical Professor of Ophthalmology Columbia University

Joseph F Rizzo III MD


Boston, MA Professor of Ophthalmology Harvard Medical School and the Massachusetts Eye and Ear Infirmary Director, Center for Innovative Visual Rehabilitation Department of Veterans Affairs

Prem S Subramanian MD PhD


Silver Spring, MD Associate Professor of Ophthalmology, Neurology, and Neurosurgery Johns Hopkins University School of Medicine Associate Professor of Surgery Uniformed Services University of the Health Sciences

No photo available

Paul H Phillips
Little Rock, AR Professor of Ophthalmology University of Arkansas for Medical Sciences

Jacinthe Rouleau MD
Montreal, QC, Canada Clinical Assistant Professor Universit de Montral

Nathan Troy Tagg MD


Iowa City, IA Neuro-Ophthalmologist Walter Reed National Military Medical Center Associate Professor of Neurology Uniformed Services University of the Health Sciences

Peter A Quiros MD
Los Angeles, CA Assistant Professor of Ophthalmology University of Southern California

R Michael Siatkowski MD
Oklahoma City, OK Professor of Ophthalmology and Vice Chair for Academic Affairs Dean A McGee Eye Institute University of Oklahoma

Roger E Turbin MD
Livingston, NJ Associate Professor of Ophthalmology and Associate Director of NeuroOphthalmology University of Medicine and Dentistry, New Jersey New Jersey Medical School

Faculty Listing

2011 Subspecialty Day

Neuro-Ophthalmology

Michael S Vaphiades DO
Birmingham, AL Professor of Ophthalmology, Neurology and Neurosurgery University of Alabama at Birmingham

Sushma Yalamanchili MD
Houston, TX Assistant Professor of Clinical Ophthalmology Weill Cornell Medical College Assistant Professor of Clinical Ophthalmology University of Texas Medical Branch at Galveston

Nicholas J Volpe MD
Chicago, IL Tarry Professor of Ophthalmology Chair, Department of Ophthalmology Northwestern University

2011 Subspecialty Day

Neuro-Ophthalmology

xi

Neuro-Ophthalmology 2011

What Is Wrong With This Picture? Recognizing the Neuro-Ophthalmic Red Flags
SATuRDAY, OCTOBER 22, 2011
6:30 AM 8:00 AM REGISTRATION/MATERIAL PICKUP/CONTINENTAL BREAKFAST Welcome and Introductions Michael S Lee MD*

Section I:

What Is Wrong With This Fundus?


Moderator: Wayne T Cornblath MD Panelists: Anthony C Arnold MD*, Larry P Frohman MD, Nicholas J Volpe MD

8:05 AM 8:09 AM 8:20 AM 8:24 AM 8:35 AM 8:39 AM 8:50 AM 8:54 AM 9:05 AM 9:09 AM 9:20 AM 9:24 AM 9:35 AM 9:39 AM 9:50 AM 9:55 AM

Case #1: Non-Arteritic Anterior Ischemic Optic Neuropathy... or Not? Discussion Case #2: Bilateral Optic Disc Edema Discussion Case #3: Unilateral Optic Disc Edema Discussion Case #4: Visual Field Loss With an Afferent Pupillary Defect Discussion Case #5: Bilateral Optic Disc EdemaPart 2 Discussion Case #6: Optic Neuritis... or Not? Discussion Case #7: Optic AtrophyWhats Next? Discussion Surgery by Surgeons REFRESHMENT BREAK and ANNUAL MEETING EXHIBITS

Michael C Johnson MD Rudrani Banik MD Byron L Lam MD* Fiona E Costello MD* R Michael Siatkowski MD* Roger E Turbin MD* Dean M Cestari MD Thomas A Graul MD

2, 16 2, 16 3, 18 3, 19 4, 19 4, 20 5, 21 6

Section II:

What Is Wrong With This Eyelid/Pupil?


Moderator: Prem S Subramanian MD PhD* Panelists: Sophia Mihe Chung MD*, Lanning B Kline MD, Jeffrey G Odel MD*, Joseph F Rizzo III MD

10:30 AM 10:34 AM 10:45 AM 10:49 AM 11:00 AM 11:04 AM

Case #8: A Dilated Pupil With Blurred Vision Discussion Case #9: Which Eyelid Is Abnormal? Discussion Case #10: The Opposite of Ptosis Discussion

Francois-Xavier Borruat MD* Marie D Acierno MD Kathleen B Digre MD

7, 23 7, 24 8, 25

* Indicates that the presenter has financial interest. No asterisk indicates that the presenter has no financial interest.

xii
11:15 AM 11:19 AM 11:30 AM 11:34 AM 11:45 AM 11:49 AM 12:00 PM

Program Schedule Case #11: Small Pupil, Droopy EyelidDiagnosis? Discussion Case #12: Large Pupil That Reacts (A Bit) Discussion Case #13: The Twitching EyelidWhats the Cause? Discussion LUNCH and ANNUAL MEETING EXHIBITS

2011 Subspecialty Day

Neuro-Ophthalmology

Michael S Vaphiades DO Reid A Longmuir MD Nathan Troy Tagg MD

8, 26 8, 27 9, 28

Section III:

What Is Wrong With This Eye Movement?


Moderator: Madhu R Agarwal MD Panelists: Eric Eggenberger DO*, Lynn K Gordon MD PhD*, Steven A Newman MD, Gregory S Kosmorsky DO

1:15 PM 1:19 PM 1:30 PM 1:34 PM 1:45 PM 1:49 PM 2:00 PM 2:04 PM 2:15 PM 2:19 PM 2:30 PM 2:34 PM 2:45 PM 2:49 PM 3:00 PM

Case #14: Fourth Nerve Palsy Discussion Case #15: Ocular Flutter Discussion Case #16: Exophoria, but Wait Thats Fatigable? Discussion Case #17: Spasmus Nutans... How Do I Know? Discussion Case #18: Slowed Saccade and a ?-Internuclear Ophthalmoplegia Discussion Case #19: Microvascular Third Nerve Palsy, but Whats Up With That Lid? Discussion Case #20: Thyroid Eye Disease... or Not? Discussion REFRESHMENT BREAK and ANNUAL MEETING EXHIBITS

Mark L Moster MD* Rod Foroozan MD

10, 30 10, 31

Gabrielle R Bonhomme MD 10, 31 Peter A Quiros MD Julie Falardeau MD Beau Benjamin Bruce MD MS* Kimberly Cockerham MD FACS* 11, 32 11, 33

11, 33

11, 34

Section IV:

What Is Wrong With This Test?


Moderator: Matthew Dean Kay MD Panelists: Valerie Biousse MD, Karl C Golnik MD*, Neil R Miller MD*, Nancy J Newman MD*

3:30 PM 3:34 PM 3:45 PM 3:49 PM 4:00 PM 4:04 PM

Case #21: Potential Sources of Error in ERG or mfERG Testing Discussion Case #22: What to Do With This Bitemporal Hemianopia Discussion Case #23: Normal ESR... Could It Still Be Giant Cell Arteritis? Discussion

Randy H Kardon MD PhD* 12, 37 Guy V Jirawuthiworavong MD Jacinthe Rouleau MD*

12, 38 13, 38

* Indicates that the presenter has financial interest. No asterisk indicates that the presenter has no financial interest.

2011 Subspecialty Day

Neuro-Ophthalmology

Program Schedule

xiii

4:15 PM 4:19 PM 4:30 PM 4:34 PM 4:45 PM 4:49 PM 5:00 PM

Case #24: Thin Retinal Nerve Fiber Layer... Really? Discussion Case #25: Homonymous Hemianopia: Wheres the Lesion? Discussion Case #26: Is a Visual Evoked Potential in Nonorganic Visual Loss Needed Here? Discussion Closing Remarks

Timothy J McCulley MD Sushma Yalamanchili MD

13, 41 13, 42

Paul H Phillips MD Andrew G Lee MD

14, 42

* Indicates that the presenter has financial interest. No asterisk indicates that the presenter has no financial interest.

Case Presentations

Section I: What Is Wrong With This Fundus?

2011 Subspecialty Day

Neuro-Ophthalmology

Section I: What Is Wrong With This Fundus?

Case #1: Non-Arteritic Anterior Ischemic Optic Neuropathy... or Not?


Michael C Johnson MD
CASE
A 76 year-old man presented 1 week after noting acute peripheral visual constriction of his left eye. His central vision was still well preserved. He did not notice any progression after onset. He denied eye pain, diplopia or episodes of transient monocular vision loss. He also denied having headaches, scalp tenderness, jaw claudication, arthralgia, myalgia, fevers, night sweats or weight loss. His past ocular history was unremarkable. His past medical history was significant for hypercholesterolemia, hypertension, coronary artery disease requiring coronary artery bypass grafting, kidney stones and hip replacement. His medications included Procardia, Toprol, Lipitor and aspirin. On examination his best corrected visual acuity measured 20/20 in each eye. He had a 1.2 log unit relative afferent pupillary defect OS. No color deficit was seen with the Ishihara color plate test. Humphrey 24-2 visual field analysis demonstrated superior altitudinal and inferior arcuate visual field defects OS and was unremarkable OD. His extraocular movements were full and no manifest eye misalignment was seen. No temporal artery tenderness or nodularity was noted. Anterior segment examination, including intraocular pressures, was normal. Fundus examination demonstrated optic disc edema OS. A cotton-wool spot was noted along the superior temporal arcade OD, but no abnormality of the optic disc was noted OD.

Case #2: Bilateral Optic Disc Edema


Rudrani Banik MD
CASE
A. 25-year-old female referred by optometrist for papilledema B. History 1. Blurry vision O.U., headaches, mild fever, and generalized malaise for 1 week 2. No tinnitus, transient visual obscurations, diplopia, pain with eye movement, or other neurologic symptoms C. Past medical history: chronic migraine headaches; morbid obesity with BMI = 37 D. Examination 1. Acuity: 20/30 O.D., 20/40 O.S.; no relative afferent pupillary defect (RAPD); HRR plates 3/6 O.U.; visual fields (VF): mild central deficits O.U. 2. Slitlamp exam: normal, with normal IOP 3. Dilated fundus exam a. Bilateral disc edema b. No retinal hemorrhages, exudates, cotton wool spots, or folds c. Normal vessels d. Few areas of hypopigmentation, ? retinochoroidal infiltrates E. Workup 1. Blood pressure 130/80 2. B-scan: no optic disc drusen 3. MRI/MRV: no mass lesions, no enhancement, normal ventricles, partially empty sella 4. Lumbar puncture: opening pressure = 15 cm H20, normal cerebrospinal fluid cell count, protein, glucose F. Patients symptoms continue to worsen

2011 Subspecialty Day

Neuro-Ophthalmology

Section I: What Is Wrong With This Fundus?

Case #3: Unilateral Optic Disc Edema


Byron L Lam MD
CASE
An asymptomatic 56yearold man was found to have unilateral optic disc edema O.S. during routine examination and was referred for evaluation. Visual acuity was 20/20 O.U. with no relative afferent pupillary defect. Humphrey visual fields were normal O.U. Funduscopic examination revealed a hyperemic swollen optic disc O.S. with blurred margins. OCT showed increased retinal nerve fiber layer thickness O.S.
Figure 1. Left eye. Figure 2. Right eye.

Goldmann and Humphrey Perimetry


See Figure 3.

Figure 1. Asymptomatic 56-year-old man with pseudoedema O.S. from unilateral anomalous optic disc.

Case #4: Visual Field Loss With an Afferent Pupillary Defect


Fiona E Costello MD
CASE Patient History
You are asked to see a 33-year-old woman for evaluation of vision loss in the left eye. Previously well. Family history is positive for MS. She denies pain. Her vision became smudged in the left eye over 48 hours. She describes seeing sparkles in the temporal visual field of the left eye.

Figure 3. Left eye.

Fundus Examination
See Figures 5 and 6.

Examination
Vision is 20/20 O.D. and 20/50 O.S. There is a left relative afferent pupillary defect. Color vision is 10/10 HRR pseudoisochromatic plates in the right and 10/10 plates in the left eye. Slitlamp examination is normal.

Figure 5.

Figure 6.

MRI
See Figures 7 and 8.

Visual Fields
See Figures 1 and 2.

Section I: What Is Wrong With This Fundus?

2011 Subspecialty Day

Neuro-Ophthalmology

macular, retina, or vitreous. The neurologic examination including motor function, reflexes, sensation, and coordination was normal. Visual fields at presentation with automated SITA-standard regimens are shown.

Figure 7.

Figure 8.

Case #5: Bilateral Disc Edema


R Michael Siatkowski MD
CASE
Child with headaches and abnormal optic discs is referred for papilledema. Exam shows 20/20 vision O.U., normal pupils, grossly normal visual fields. Optic discs elevated with anomalous vessels; spontaneous venous pulsations are absent.
Figure 1.

Case #6: Optic Neuritis... or Not?


Roger E Turbin MD
CASE Patient History
In 2002, a 30-year-old Cuban man complained of 3 days of blurry vision in the right eye, ipsilateral pain with eye movement, and discomfort radiating from the right eye to his temple. The patient had undergone internal fixation of a tibia injury. He was otherwise healthy, working as a bartender with a pack-per-day smoking habit. He had no common neurologic complaints to suggest previous demyelinating episodes.

Physical Examination
BCVA was 20/20- in the affected right eye, decreasing to 20/40 within 3 days (6 days after symptom onset), and 20/15 on the left. Eight of 12 and 12/12 Ishihara pseudoisochromatic plates were correctly identified on the right and left, respectively. Pupils were symmetric, reactive to light, and a right 1.2 log unit relative afferent pupil defect was present. Ocular motility examination was normal, the eyes were orthophoric, and no pathologic nystagmus was present. The slitlamp biomicroscopic examination was unremarkable. The right optic nerve, initially flat and hyperemic at presentation, developed optic disc edema with mild 360 nerve elevation and nerve fiber layer edema by the third day. The left optic nerve was normal, with 0.25 by 0.2 cup, normal color and nerve fiber layer. No abnormalities were detected of the

Figure 2.

2011 Subspecialty Day

Neuro-Ophthalmology

Section I: What Is Wrong With This Fundus?

Within the first few days, the visual field in the right eye worsened.

Figure 3.

Figure 5.

What evaluation should be obtained to support the likely diagnosis? Fat suppressed, contrast enhanced images of the orbit and fluid attenuated inversion recovery (FLAIR) images of the brain show enhancement of the intraorbital optic nerve and sheath as well as scattered foci of abnormal T2 signal in the right frontal lobe without enhancement.

Intravenous methyl prednisone, followed by an oral course of prednisone, based on the optic neuritis treatment trial protocol regimen, was offered. The patient chose to undergo the equivalent high-dose therapy in oral form for financial reasons. His visual acuity, visual field, dyschromatopsia, and optic disc swelling normalized. Now lets fast-forward in time.... Three weeks after the prednisone was tapered, ipsilateral pain returned, vision decreased to counting fingers, and visual field dramatically worsened in the affected eye. What is wrong with the original diagnosis? What additional evaluation is indicated?

Case #7: Optic Atrophy Whats Next?


Dean M Cestari MD
CASE
A 74-year-old man presents to his primary ophthalmologist complaining of 8 months of blurred vision O.S. He denies pain and any additional symptoms. His past medical history is significant for hypertension, diabetes, and erectile dysfunction. His medications include lisinopril, macrophage, and Viagra p.r.n. His examination demonstrates visual acuity (VA) 20/20 O.D. and 20/40 O.S. Slitlamp exam is significant for 1+ cataracts O.U. On Ishihara color plates he is 8/8 O.D. and 6/8 O.S. There is a subtle relative afferent pupillary defect O.S. IOPs are 21 mmHg O.U. The right optic nerve appears normal, with a cup/disc of 0.1. The left optic nerve has a cup/disc of 0.4 with pallor. There is a funny-looking vessel on the left optic disc. Humphrey visual fields are full O.D. and demonstrate a central scotoma O.S.

Figure 4.

2011 Surgery by Surgeons Update

2011 Subspecialty Day

Neuro-Ophthalmology

2011 Surgery by Surgeons Update


Thomas A Graul MD

With this years passage of legislation in Kentucky that allows optometrists to perform laser surgery, the American Academy of Ophthalmologys partnership with ophthalmic subspecialty and state societies on the Surgery by Surgeons campaign becomes even more important in protecting quality patient eye care across the country. In 2009-2010, the Eye M.D.s serving on the Academys Secretariat for State Affairs collaborated with the leadership of many state ophthalmology societies on legislative battles in which optometry continued to push for expanded scope of practice. Leadership of subspecialty societies provided essential support in some of these battles. Success was reached in Idaho, Maine, Mississippi, Nebraska, South Carolina, Texas, Washington and West Virginia, where surgery provisions were removed and/or bills were defeated. In 2011, the stakes were raised with the disappointing outcome in Kentucky. The Kentucky legislation also includes the creation of an independent optometric board; no other board or state agency has the authority to question what constitutes the practice of optometry. The Secretariat for State Affairs continues to work diligently with state society leaders in South Carolina, Nebraska, Tennessee and Texas to ensure that a Kentucky outcome is not repeated elsewhere. For example, following the passage of legislation in Kentucky, fundraising material by organized optometry in Tennessee made it clear that they would like to replicate optometrys outcome in Kentucky and have begun discussions with state legislators. The Surgical Scope Fund (SSF) is a critical tool of the Surgery by Surgeons campaign to protect patient quality of care. The Academy relies not only on the financial contributions via the SSF by individual Eye M.D.s but also on the contributions made by ophthalmic state, subspecialty and specialized interest societies. Many subspecialty societies contributed to the SSF in 2010 and 2011, and the Academy counts on their continued support. The results in Kentucky should be viewed as a failure neither of the SSF nor of the Academys Secretariat for State Affairs, who geared up immediately to strategize with Kentucky Academy physician leadership. In a period of 15 days, with no advanced warning, optometry was able to introduce and pass a bill in the Kentucky state legislature and secure its passage into law. An SSF disbursement actually assisted with critical media buys and powerful public messaging favoring ophthalmology and quality patient eye care for the citizens of Kentucky. This should be a lesson to each Eye M.D. in the country about the importance of contributions to your state eyePAC and to the SSF. Leaders of the North American Neuro-Ophthalmology Society (NANOS) are part of the Academys Ophthalmology Advo-

cacy Leadership Group (OALG), which has met for the past four years in the Washington DC area to provide critical input and to discuss and collaborate on the Academys advocacy agenda. NANOS remains a crucial partner to the Academy in its ongoing federal and state advocacy initiatives. As a 2011 Congressional Advocacy Day (CAD) partner, NANOS ensured a strong presence of neuro-ophthalmology specialists to support ophthalmologys priorities as over 350 Eye M.D.s had scheduled CAD visits to members of Congress in conjunction with the Academys 2011 Mid-Year Forum in Washington DC. At the state level, the Academys Surgery by Surgeons campaign has demonstrated a proven track record. Kentucky was an outlier; the Academys SSF has helped 31 state ophthalmology societies reject optometric surgery language. Help us help you protect our patients and quality eye care. The Academys SSF remains a critical tool in the Surgery by Surgeons campaign. The SSF Committee works hard on your behalf to ensure the ongoing strength and viability of the SSF. Thomas Graul MD (Nebraska): Chair Arezio Amirikia MD (Michigan) Kenneth P Cheng MD (Pennsylvania) Bryan S Lee MD PhD (Maryland): Consultant Richard G Shugarman MD (Florida) Stephanie J Marioneaux MD (Virginia) Bryan S Sires MD PhD (Washington) Andrew Tharp MD (Indiana) Ex-officio members: Cynthia A Bradford MD Daniel J Briceland MD The SSF is our collective fund to ensure that optometry does not legislate the right to perform surgery. Do not forget about Congress; ophthalmologys influence is expressed through OPHTHPAC. Just as a strong state presence is needed, so do we need to remain strong in the federal arena. While OPHTHPAC is the third largest medical PAC, a mere 15% of the Academys membership contribute. The Kentucky legislation is not in the best interests of patient safety and quality patient care. Ophthalmology needs the active support of every memberand this includes contributions to the SSF, state eye PACs and OPHTHPAC. Please respond to your SSF Committee and OPHTHPAC Committee colleagues when they call on you and your subspecialty society to contribute. There are some decisions that require thought, but donating $500 to the SSF and OPHTHPAC is the easy answer for you and your patients. Do it today. Do it now.

2011 Subspecialty Day

Neuro-Ophthalmology

Section II: What Is Wrong With This Eyelid/Pupil?

Section II: What Is Wrong With This Eyelid/Pupil?

Case #8: A Dilated Pupil With Blurred Vision


Franois-Xavier Borruat MD
CASE
The degree of anisocoria was maximal in bright light and decreased in darkness. Slitlamp examination was unremarkable except for a round but completely nonreactive mydriasis of the left pupil. The left pupil reacted to neither bright light nor attempted convergence.

The patient denied any use of eye drops, has not been gardening (driving for 10 hours), but admitted using scopolamine cutaneous patch for motion sickness. Exhausted after her long trip and the car accident, she went immediately to bed after removing her scopolamine patch. She woke up with mydriasis and accommodation palsy of her left eye, most likely due to direct finger-toeye contamination. After one week, she was asymptomatic.

Figure 4.

Figure 1.

Case #9: Which Eyelid Is Abnormal?


Marie D Acierno MD
CASE
54-year-old male with new onset headache. He believes that his left eye is wider during the dull headache episodes. He also complains of dull, intermittent cervical pain for 1 week. His past medical history is significant for hypertension managed with Dyazide. He is a nonsmoker. VA 20/20 O.U. Full ocular ductions O.U.; no nystagmus or ocular misalignment Confrontation visual fields O.U.

External Examination
Figure 2.

Pilocarpine 2% was instilled in both eyes and induced a significant miosis of the right pupil whereas the left pupil was not affected.

No proptosis, enhanced ptosis, eyelid lag, lagophthalmos, or fatigable ptosis; subtle left eyelid retraction. Margin-to-reflex distance (MRD) 1: O.D. 3.5 mm; O.S. 6mm MRD 2: O.D. 3.5 mm; O.S. 5 mm PF: O.D. 7 mm; O.S. 11 mm LF: O.D. 15 mm; O.S. 14 mm

Biomicroscopic Examination
Pupils Dim: O.D. 3.5; O.S. 5.0 Bright: O.D. 3.0; O.S. 4.0
Figure 3.

Anterior and posterior segment examination unremarkable.

Section II: What Is Wrong With This Eyelid/Pupil?

2011 Subspecialty Day

Neuro-Ophthalmology

Questions
What testing would you do next in the clinic/office setting? What tests do you want to order? What is your differential diagnosis?

Case #11: Small Pupil, Droopy Eyelid... Diagnosis?


Michael Vaphiades DO
CASE
A 49-year-old woman with a history of migraine headaches and long-term contact lens use was noted to have anisocoria and ptosis. She was referred to rule out painful Horner syndrome O.S. The patient came with a contrasted cranial MRI, MRA head and neck, and chest CT scan, all of which were normal. On examination the acuity was 20/20 O.U., color was 15/15 O.U., pupils measured 7 mm O.D. and 5 mm O.S. in dim light; and 4 mm O.D. and 3 mm O.S in bright light. No relative afferent pupillary defect. After instillation of 10% cocaine drops O.U., both pupils dilated. Palpebral fissures measure 12 mm O.D. and 11 mm O.S. The ocular motility was normal, as was the fundus in each eye.

Case #10: The Opposite of Ptosis


Kathleen B Digre MD
CASE
A 21-year-old male is referred for visual changes. His complaint was things looked funny, including blurring, especially with reading, perhaps a mild diagonal diplopia, and headache. He was seen overseas while on a mission and diagnosed with possible MS. No significant past medical history. Family history positive for aneurysm. Examination: 20/20 O.U. He had bilateral eye lid retraction. Pupils 7.5 mm O.U. in light and 9 mm O.U. in darkness. While the light reaction was present, it was very slow. Near reaction was better than light reaction. No relative afferent pupillary defect was present. Visual fields were full to confrontation. Extraocular muscles showed limited upgaze, and there was convergence retraction nystagmus. Funduscopic examination was normal. An MR with contrast was ordered.

Case #12: Large Pupil That Reacts (ABit)


Reid Longmuir MD
A 53-year-old female presents with a chief complaint of a dilated left pupil and associated glare. She complains that light coming in is too bright and is causing blur. Her past medical history is unremarkable. Her past ocular history is remarkable for a history of Fuchs endothelial dystrophy, for which she has undergone penetrating keratoplasty on the right eye, and Descemetstripping endothelial keratoplasty on the left. She denies using any red-cap drops lately, but she has used them in the past. When asked if the pupil problem began before or after surgery, the patient reports that she doesnt know, stating that the glare was far worse before the surgery but it is still there. She hadnt noticed anything about her pupil until a friend pointed it out last week. She denies diplopia or ptosis. She describes a dull ache over the left eye that is aggravated by bright light. Exam reveals 20/40 vision in each eye. IOP is 12 O.D., 17 O.S. External examination reveals normal lid position bilaterally. No ocular misalignment is noted, with full ocular motility O.U. Examination of the pupils reveals O.D. 4 mm dark, 2 mm light; O.S. 6 mm in dark and light. No afferent pupillary defect is noted. There is no near response in the left pupil. Slitlamp examination shows clear corneal graft O.D. and clear endothelial graft O.S., otherwise only remarkable for cataract. Funduscopic examination is normal. Workup includes administration of dilute pilocarpine (0.1%), which yields no response in either eye. 1% pilocarpine is then administered, and the right pupil constricts dramatically. The left pupil shows virtually no response. What should be the next step? A. MRI/MRA brain to view the left third nerve and posterior communicating artery B. Record review to look at pupils before and after surgery O.S.

Figure 1.

What is your differential diagnosis? a. b. c. d. thyroid eye disease normal eye exam proptosis with tumor brainstem disorder

What tests would you order?

2011 Subspecialty Day

Neuro-Ophthalmology

Section II: What Is Wrong With This Eyelid/Pupil?


5. Urrets-Zavalia A. Fixed dilated pupil, iris atrophy and secondary glaucoma: a distinct clinical entity following penetrating keratoplasty for keratoconus. Am J Ophthalmol. 1963; 56:257-265. 6. Tuft SJ, Buckley RJ. Iris ischaemia following penetrating keratoplasty for keratoconus (Urrets-Zavalia syndrome). Cornea 1995; 14(6):618-622.

C. Medication history to look for source of pharmacologic dilation O.S. D. Wait 1 month to allow supersensitivity to cholinergics to develop, then repeat pilo test

Comments
This case highlights the potential pitfalls in trying to diagnose Adies tonic pupil versus other causes of a dilated or minimally reactive pupil. The most worrisome cause of unilateral dilated pupil is third nerve palsy, but this patient does not have ptosis or strabismus, making this diagnosis very unlikely. Pharmacologic dilation could result in an examination like this, including pilocarpine testing, and it is always wise to consider the possibility. This patient has a history of use of mydriatics but denies it recently. Another consideration is that the pupil is dilated due to her surgical history O.S. This is well described in anterior segment surgery, particularly corneal procedures. Named UrretsZavalia syndrome, this condition was first described in 1963 in a patient with an atonic pupil after penetrating keratoplasty. It has subsequently been described in numerous other anterior segment procedures, including Descemet-stripping endothelial keratoplasty. In those cases, the IOP was felt to be a key contributor leading to iris ischemia, and therefore the patients perioperative records for the left eye would be key to a final diagnosis once the pilocarpine testing is negative. It is important to note that Adies tonic pupil progresses in stages, from a tonic dilated pupil to a partially reinnervated sphincter muscle with light-near dissociation. The newly tonic pupil may not have immediate supersensitivity to dilute pilocarpine, so a negative test for Adies may have to be repeated. It is also important to look for other clues that vote for or against Adies. Segmental sphincter function is classic for Adies. Lightnear dissociation is classic once reinnervation with accommodative fibers has taken place. Adies is frequently bilateral, so careful examination of the contralateral pupil for light-near dissociation and segmental deinnervation is also helpful. Conversely, transillumination defects or synechiae may suggest prior trauma, surgery, or inflammation. These exam findings are also typical of previous episodes of angle closure or other sources of very high IOP. A previously performed peripheral iridotomy would be informative in that regard. Iridocorneal endothelial syndrome, or ICE syndrome, can lead to a unilateral atonic (or nearly atonic) pupil. This condition is recognized with careful anterior segment and gonioscopic examination. In its early stages, particularly in the essential iris atrophy subgroup, a patient may present with anisocoria rather than raised IOP.

7. Maurino V, Allan BD, Stevens JD, Tuft SJ. Fixed dilated pupil (Urrets-Zavalia syndrome) after air/gas injection after deep lamellar keratoplasty for keratoconus. Am J Ophthalmol. 2002; 133(2):266268. 8. Niknam S, Rajabi MT. Fixed dilated pupil (Urrets-Zavalia syndrome) after deep anterior lamellar keratoplasty. Cornea 2009; 28(10):1187-1190. 9. Fourni P, Ponchel C, Malecaze F, Arn JL. Fixed dilated pupil (urrets-zavalia syndrome) and anterior subcapsular cataract formation after Descemet stripping endothelial keratoplasty. Cornea 2009; 28(10):1184-1186. 10. Shields MB. Axenfeld-Rieger and iridocorneal endothelial syndromes: two spectra of disease with striking similarities and differences. J Glaucoma. 2001; 10(5 suppl 1):S36-38.

Case #13: The Twitching Eyelid Whats the Cause?


Nathan T Tagg MD
CASE
A 45-year-old man was seen by his physician for 15 days of frequent, intermittent twitching of the right lower eyelid. Over the next few days the twitching spread to involve the entire right side of the mouth and right side of the neck. He had no other visual or neurological complaints. Examination demonstrated normal visual function, normal ocular motility, and normal pupillary function. Examination of the eyelids and face revealed abnormal eyelid and facial movements on the right. There was no facial weakness. The remaining neurological exam was normal. The video presentation shows continuous undulating and rippling movements of the right side of the face around eyelids, the right cheek, angle of mouth, and platysma. These findings are consistent with facial myokymia. Diagnostic evaluation included an MRI scan of the brain with contrast, which demonstrated a right-sided lesion at the level of the caudal pons.

Selected Readings
1. Kawasaki A. Physiology, assessment, and disorders of the pupil. Curr Opin Ophthalmol. 1999; 10(6):394-400. 2. Lee AG, Taber KH, Hayman LA, Tang RA. A guide to the isolated dilated pupil [review]. Arch Fam Med. 1997; 6(4):385-388. 3. Kardon RH, Corbett JJ, Thompson HS. Segmental denervation and reinnervation of the iris sphincter as shown by infrared videographic transillumination. Ophthalmology 1998; 105(2):313-321. 4. Jacobson DM, Vierkant RA. Comparison of cholinergic supersensitivity in third nerve palsy and Adies syndrome. J Neuroophthalmol. 1998; 18(3):171-175.

10

Section III: What Is Wrong With This Eye Movement?

2011 Subspecialty Day

Neuro-Ophthalmology

Section III: What Is Wrong With This Eye Movement?

Case #14: Fourth Nerve Palsy


Mark L Moster MD
CASE
A 41-year-old woman presents with 4 months of diplopia and worsening of her prior headaches. The diplopia is binocular and vertical. It initially began only with left head tilt and progressed to other positions. It is worst in downgaze, right gaze, and left head tilt and improved with right head tilt. Neurologic consultation revealed a normal examination, and MRI showed a few nonspecific white matter lesions, which raised the possibility of demyelinating disease. She was referred for neuro-ophthalmologic consultation. Past medical history was significant for migraine and hypothyroidism. Medications were sumatriptan (Imitrex), levothyroxine (Synthroid), and lansoprazole (Prevacid). Neuro-ophthalmologic examination revealed a normal afferent examination. There was a spontaneous right head tilt. Ductions were full, but she had a left hypertropia on alternate cover test, measuring 5 prism diopters (PD) in primary position, 1 PD in left gaze, 12 PD in right gaze, 2 PD with right head tilt, 18 PD with left head tilt, 1 PD in upgaze and 16 PD in downgaze. Vertical fusional capacity measured 2 PD.

Case #16: Exophoria, but Wait Thats Fatigable?


Gabrielle Bonhomme MD
CASE Chief Complaint
A previously healthy, 27-year-old Liberian man presents to the office reporting 3 days of binocular diplopia. These symptoms of diplopia are intermittent, occur during mid-afternoon classes, and prevent him from studying for exams, given his difficulty reading at night. He quickly develops a dull headache while studying.

History
His past medical history is significant only for seasonal allergies, treated with OTC meds. On ocular history, he denies any known strabismus, periorbital trauma, or previous episodes of diplopia. Family history is unknown. Social history: He is a graduate student. Medications: None Allergies: No known drug allergies

Case #15: Ocular Flutter


Rod Foroozan MD
CASE
42-year-old male presented with jumping images and abnormal eye movements for 6 days. He was in his usual state of good health until 6 days prior to presentation, when he noted involuntary flicking of his eyes while working at the computer. Five days prior to presentation he developed severe nausea and vomiting, had difficulty walking, and could not ambulate without falling. He had a prior history of tinnitus in his right ear, weight loss, and cold sores of the lips. He saw an ophthalmologist, who thought he had nystagmus, and an MRI of the brain with contrast was normal. The remainder of the eye examination was normal. First video shows ocular flutter, with horizontal back-to-back saccades. Patient was hospitalized and a number of tests were performed, including whole body imaging, lumbar puncture, and serology. He was found to have positive serology indicative of recent infection to coxsackie B virus. He had a slowly progressive, spontaneous improvement of the abnormal ocular oscillations over the ensuing months. Second video shows normal eye movements months after initial examination.

Examination
Visual acuity: 20/20 bilaterally IOP: 15 bilaterally Ishihara: 11/11 bilaterally Pupils: No afferent pupillary defect, no anisocoria Fields by confrontation are full bilaterally Slitlamp of the anterior segment and dilated funduscopic exam are normal. Motility and alignment examination reveal a variable exophoria with bilateral adduction deficit. External examination reveals 2-mm relative left ptosis. What is your diagnosis?

Figure 1.

2011 Subspecialty Day

Neuro-Ophthalmology

Section III: What Is Wrong With This Eye Movement?

11

Case #17: Spasmus Nutans... How Do I Know?


Peter A Quiros MD
CASE
A 16-month-old full-term child, product of a gravida 1, para 1 uneventful pregnancy with spontaneous vaginal delivery, presents with 1 week of abnormal, intermittent headshaking and eye movements. The child is able to x and follow a 2.5-cm target at 65 cm. Ductions and versions are full and is orthophoric throughout testing. External exam and dilated funduscopic exam are normal. The head movements are best described as head shaking or bobbing. The abnormal eye movements are best described as an asymmetric high-frequency, low-amplitude horizontal jerk nystagmus that is best seen in eccentric left gaze (a video accompanies the presentation). The child is otherwise neurologically intact with a normal, nonfocal neurological examination. No neuro-imaging has been performed.

Case #19: Microvascular Third Nerve Palsy, but Whats Up With That Lid?
Beau Benjamin Bruce MD MS
CASE
A 58-year-old woman presents with a 2-day history of right brow pain followed by the development of diplopia and right ptosis. She has a past history of hypertension and diabetes mellitus, both adequately controlled with oral medications. She presents to her ophthalmologist. Her examination is notable only for complete right-sided ptosis, normally reactive pupils, and complete limitation of supra-, infra-, and adduction of the right eye. The actions of the fourth and sixth cranial nerves are intact. An erythrocyte sedimentation rate and C-reactive protein are normal. A MRI study and magnetic resonance angiogram of the brain without contrast are obtained and read as normal. She is told that this is likely a microvascular third cranial nerve palsy and that she will probably improve within the next 3 months. Follow-up is arranged for that time. She is unable to keep her originally scheduled follow-up visit, but returns 5 months later. At her follow-up visit, she still has some ptosis and extraocular motility limitation consistent with incomplete recovery of a right third cranial nerve palsy. However, you also note new findings: when she looks down, the right eyelid does not lower fully, and when she adducts her right eye, her ptosis improves.

Case #18: Slowed Saccade and a ? Internuclear Ophthalmoplegia


Julie Falardeau MD
CASE
A 77-year-old patient with a past history of pernicious anemia and hypothyroidism presented with a 5-day-history of painless, binocular diplopia, worse in right gaze. His diplopia was preceded by a few episodes of transient blurred vision. He denies any other ophthalmic or neurological symptoms. He denies any temporal pain, jaw claudication, scalp tenderness, or any other symptoms suggestive of temporal arteritis. His examination showed visual acuity of 20/25 in both eyes with normal color vision. He has no anisocoria or relative afferent pupillary defect. External examination is unremarkable. Anterior segment is only remarkable for mild nuclear sclerosis in both lenses. Dilated funduscopic evaluation is normal. On ocular motility testing, he appears to have full range of extraocular movements bilaterally. His saccades appear normal in his right eye, but he has slowed adducting saccade in his left eye. A mild exophoria superimposed with 2 prism diopters (PD) of right hypertropia is present in primary position, with 8 PD of exotropia and 2 PD right hypertropia in right gaze. He has 2 PD of right hypertropia in left gaze. A diagnosis of left internuclear ophthalmoplegia (INO) is made. MRI of the brain is ordered and shows no lesion in the medial longitudinal fasciculus. The patient returns 1 month later for reassessment. He now has a right ptosis as well as right upgaze deficit.

Case #20: Thyroid Eye Disease... or Not?


Kimberly Cockerham MD FACS
CASE
A 55-year-old salesman presented with new-onset double vision. His past medical history was remarkable for Graves disease. On examination, he had 20/20 vision, normal color testing, HVF and OCT. He had 30 mm of proptosis, increased resistance to retropulsion, and an IOP of 25 O.U. in primary gaze. He had 2 mm of upper and lower eyelid retraction with lid lag. His anterior segment was remarkable for mild inferior punctuate keratopathy. His posterior segment was normal.

12

Section IV: What Is Wrong With This Test?

2011 Subspecialty Day

Neuro-Ophthalmology

Section IV: What Is Wrong With This Test?

Case #21: Potential Sources of Error in ERG or mfERG Testing


Randy Kardon MD PhD
CASE Patient Present Illness
A 67-year-old woman was referred to a retina specialist because of flashes of light noticed in the last 2 weeks and increase in sensitivity to light. She was not sure if the flashes were coming from one or both eyes, but they were happening almost all of the time, and appeared as numerous gold-colored sparklers going off in her vision, more noticeable in dim light or with her eyes closed.

Neuro-ophthalmology Consult
The neuro-ophthalmology exam showed constricted visual fields on automated perimetry (which was confirmed on confrontation testing), normal pupil exam, normal ocular motility, and a normal anterior chamber and fundus exam. The patient was photophobic during the indirect ophthalmoscope exam and had difficulty keeping her eyes open, with constant blinking. The combination of constricted visual field and constant photopsias raised the suspicion of a retina problem, in spite of a normal appearing retina and prior report of normal testing by OCT and fluorescein angiography. A Ganzfeld electroretinogram were obtained. The Ganzfeld ERG results were reported to be normal. A second SD-OCT was obtained of the optic nerve, retinal nerve fiber layer, and macula. On careful inspection, it appeared that the high-density reflective line at the junction of the inner and outer segments of the B-scan on OCT was disrupted. The ERG results, including the actual tracings that were recorded, were then carefully reviewed.

Medical History
Osteoporosis, gastric reflux disease, and right arm edema in the last few months. She had two previous transient ischemic attacks (TIAs), with vertigo and facial numbness on one occasion and transient hearing loss on another.

Medicines
Aspirin, alendronate (Fosamax), and estrogen replacement

Case #22: What to Do With This Bitemporal Hemianopia


Guy V Jirawuthiworavong MD
CASE

Exam Findings
Vision: BCVA to 20/20 O.D. and O.S. Pupillary exam and ocular motility were reported to be normal. Anterior chamber exam showed a normal cornea and adnexa, no signs of anterior chamber inflammation, and pseudophakic O.U. The retina exam was completely normal, with normal vessel caliber, no pigmentary changes, and no signs of posterior uveitis. A fluorescein angiogram was obtained by the retina specialist, which was normal. In addition, spectral domain OCT (SD-OCT) was also obtained of the macula and was reported to be normal. Because of the normal retina exam, including normal OCT and fluorescein angiography, the complaint of photopsias and the prior history of TIAs, the retina specialist referred the patient to a neurologist.

Neurology Consult
The neurological exam was normal, and a contrast-enhanced MRI of the brain and MRA were obtained, which were normal. Visual evoked potentials (VEP) performed by the neurologist showed no abnormal slowing of the evoked cortical responses. The patient was then referred to a neuro-ophthalmologist for further evaluation.

White, 47-year-old female presents with visual loss for more than 24 hours. On the evening of April 26, 2010, patient complained of having nausea, stomach cramps, and diarrhea that kept her up all night going to the bathroom. She did not feel her vision was compromised as she had gotten up frequently to go to the bathroom. The following morning, April 27, she woke up at 8:00 AM and could not read the clock in her room. She had an appointment with her chiropractor and decided to keep it. She had difficulty seeing the car speedometer and street signs on her way to and from the chiropractors office. Her vision also had a blue tinge to it. She felt that her central vision was affected more than her peripheral vision. There was a big splotch in the center that changed from pink to red to gray to black. By 3:00 PM, she was so exhausted from not sleeping well the night before that she decided to go to sleep early. She woke up on April 28 and noted no improvement in her vision. In a panic, she rushed to the hospital with her sister as her driver. The emergency room performed a MRI of the brain which was read as IMPRESSION: No acute intracranial finding. Past ocular history was significant for visible optic disc drusen O.D. diagnosed in 2007. Her past medical history includes chronic fatigue syndrome and headaches. She takes pain relievers as needed for chronic back pain. She recently quit smoking after 31 years and occasionally has a drink of alcohol. She was unaware of any family members who were blind or anyone who had multiple sclerosis.

2011 Subspecialty Day

Neuro-Ophthalmology

Section IV: What Is Wrong With This Test?

13

She denied recent weight loss, malaise/fatigue or weakness other than what she experienced 2 nights prior, and no cold or heat intolerance. She did not have any pain with eye movement, double vision, or headaches. She denied gait difficulties and numbness or tingling of her hands, feet, or face. There was no change in menstruation, and she did not have galactorrhea. On examination, her visual acuity was O.D. 20/200 and O.S. 20/400 and color vision was 0/14 O.U. There was no relative afferent pupillary defect. Versions and ductions were full, and there was no nystagmus. She was orthophoric. Optokinetic nystagmus testing revealed normal saccades O.U. Confrontational fields were normal to count fingers in the periphery; however, patient had difficulty seeing centrally as noted with red desaturation testing. The slitlamp exam was normal. Dilated fundus exam showed visible disc drusen of the right optic nerve. The left optic nerve appeared pink with sharp borders and without peripapillary atrophy. The macula exam and periphery seemed unremarkable. Visual field testing was completed the following day. Central bitemporal hemianopic visual field defect was noted. Further diagnostic testing was performed.

Case #24: Thin Retinal Nerve Fiber Layer... Really??


Timothy James McCulley MD
CASE
A 62-year-old woman presents with a BCVA of 20/50 O.U. Following uneventful cataract extraction O.D., BCVA is measured at 20/20 O.D. and 20/70 O.S. The cataract in the left eye is not felt to have progressed to a degree accountable for the decline in visual acuity. An OCT is obtained. The retinal nerve fiber layer (RNFL) appears asymmetric, and the patient is sent for neuroophthalmic evaluation. On examination BCVA is confirmed to be 20/20 O.D. and 20/70 O.S. There is no relative afferent pupillary defect. The optic nerves appear healthy and symmetric, with no atrophy or edema. Findings on automated perimetry are normal on the right and demonstrate mild diffuse depression O.S., with no significant pattern deviation. Possible explanations for and implications of the OCT findings will be explored.

Case #23: Normal ESR... or Not? Could It Still Be Giant Cell Arteritis?
Jacinthe Rouleau MD
CASE
A 78-year-old female presents with acute visual loss O.S. Her past ocular history is positive only for bilateral cataract surgery 10 years ago. She has a past medical history significant for hyperlipidemia, hypertension, and osteoarthritis. Two days ago, she complained to her daughter about dimming of light lasting 2-3 minutes in her left eye. Today, she awoke with severe visual loss O.S., and there has been no improvement since. She has had frontal headache for 1 week. Her weight is stable and she denies jaw claudication or scalp tenderness. On examination, visual acuity is 20/20 (6/6) O.D. and light perception O.S. A relative afferent pupillary defect is present O.S. Goldmann visual field is normal O.D. and not testable O.S. Anterior segment exam reveals pseudophakia O.U. Dilated fundus examination shows a normal optic nerve O.D. with a cup/disc ratio of 0.4. The optic nerve O.S., however, is pale and edematous. The macula and peripheral retina are normal O.U. Her temporal arteries on both sides are pulsatile, non-tender, and normal in appearance. Because of your suspicion for giant cell arteritis, you order an erythrocyte sedimentation rate (ESR) and it returns at 30 mm/ hour. How should you manage this case at this point?

Case #25: Homonymous Hemianopia: Wheres the Lesion?


Sushma Yalamanchili MD
CASE
History of present illness: 69-year-old white male with a past medical history of hypertension with difficulty driving home and parking in the garage. Chief complaint: I knocked the right side mirror of my car while parking in the garage. He has been having problems with reading and driving for the past year. The patients wife said that he has difficulty locating and identifying his medication and glasses. She says that even though his glass of water is right in front of him, he has a hard time finding it. She took him to an optometrist last year because he was complaining that he couldnt see well. He even stopped paying the bills. His vision was found to be 20/20 in both eyes. Then, when he hit the side of the garage, she took him to a local ophthalmologist. His vision was fine, but he had a field defect and was referred to neuro-ophthalmology. Review of systems: Otherwise negative Past medical history: Hypertension Past surgical history: Tonsillectomy Past ocular history: No glaucoma, injuries, or surgeries Allergies: Amoxicillin (hives) Medications: Lisinopril 10 mg/day Visual acuity was 20/25 in the right eye and 20/20 in the left eye. Extraocular movements were full. There was no afferent papillary defect. Slitlamp exam was within normal limits. Dilated fundus exam was within normal limits. No disc edema or pallor. Visual field exam shows a right homonymous hemianopia. MRI is unremarkable.

14

Section IV: What Is Wrong With This Test?

2011 Subspecialty Day

Neuro-Ophthalmology

Case #26: Is a Visual Evoked Potential in Nonorganic Visual Loss Needed Here?
Paul H Phillips MD
CASE
A 26-year-old, previously healthy man complained of sudden visual loss in his right eye after blunt head trauma from a motor vehicle accident. The head trauma did not result in loss of consciousness. Emergency room records on the day of the accident indicated no external signs of injury and a normal head computed tomography without contrast. He claimed that the visual loss in his right eye made it impossible to continue his occupation as a truck driver and was applying for complete disability. Ophthalmological examination revealed a visual acuity of 20/100 O.D., 20/20 O.S. Confrontation visual fields showed constriction O.D. and were full O.S. Pupils were equal and reactive, with no relative afferent pupillary defect. Ocular motility, external, slitlamp, and funduscopic examination were normal. Functional visual loss was suspected and further testing was performed. Visual acuity testing with polarized Snellen chart and glasses showed improvement of his vision O.D. to 20/50. Titmus stereoacuity was intact within 40 seconds of arc. Goldman visual field testing of the right eye showed constriction within 10 degrees of fixation O.D. and was full O.S. Tangent visual field testing O.D. showed tubular fields with 10 degrees of constriction with the target at 1 meter and 2 meters. Pattern visual evoked response (VER) testing showed no response O.D. and a normal response O.S. Despite multiple clinical indications of functional visual loss, the VER seemed to indicate an organic etiology of this patients visual loss O.D. A statement is required from the physician regarding the patients ability to continue his profession as a truck driver.

Answer and Teaching Points

16

Section I: What Is Wrong With This Fundus?

2011 Subspecialty Day

Neuro-Ophthalmology

Section I: What Is Wrong With This Fundus? Answer and Teaching Points
Case #1: Non-Arteritic Anterior Ischemic Optic Neuropathy... Or Not?
Michael C Johnson MD
Answer and Teaching Points
Arteritic anterior ischemic optic neuropathy (AAION) presenting with pallid optic disc edema in the affected eye, cotton-wool spots in the contralateral eye and no systemic symptoms, occult giant cell arteritis. His ESR, CRP and platelet count were elevated at 73 mm/h, 22.8 mg/L, and 531 X 109/L respectively. A left temporal artery biopsy was positive for giant cell arteritis.
3. Hayreh SS, Podhajsky PA, Zimmerman B. Ocular manifestations of giant cell arteritis. Am J Ophthalmol. 1998;125(4):509-20. 4. Beck RW, Savino PJ, Repka MX, et al. Optic disc structure in anterior ischemic optic neuropathy. Ophthalmology 1984;91(11):13347. 5. Melberg NS, Grand MG, Dieckert JP, et al. Cotton-wool spots and the early diagnosis of giant cell arteritis. Ophthalmology 1995;102(11):1611-4.

Case #2: Bilateral Optic Disc Edema


Rudrani Banik MD
Answer and Teaching Points

Teaching Points
Giant cell arteritis (GCA) is the most common primary vasculitis of adults in the Western world.1 The most common cause of permanent visual loss due to GCA is arteritic anterior ischaemic optic neuropathy (AAION). Among patients with AAION, 25-50% will suffer a similar event in the contralateral eye if left untreated, typically within 1-14 days.2 It is therefore imperative to distinguish between AAION and non-arteritic anterior ischemic optic neuropathy (NAION), so that proper treatment of GCA is not delayed. If there is a past medical history of cardiovascular disease and the typical constitutional symptoms of GCA are lacking, distinguishing between AAION and NAION based on history alone may be very difficult. However, on examination, any of the following features may be considered to be highly suggestive of AAION: severe pallid optic disc edema (often described as chalk white disc swelling), disc swelling in combination with retinal ischemic lesions (central retinal artery occlusion, cilioretinal artery occlusion, or cotton-wool spots), and a normal or large optic cup in the contralateral eye.2-4 Retinal ischemia manifesting with cotton-wool spots alone can also be a presenting sign of GCA.5 Retinal ischemic lesions in combination with ischemic optic neuropathy in a non-diabetic patient indicates involvement of two different vascular territories of the eye (the central retinal artery and the posterior ciliary artery circulation) and GCA must be seriously considered in these cases. This case illustrates several of the features that may be used to distinguish AAION from NAION on examination. This can be particularly helpful when the patient history is non-contributory or confounding and can help guide the ophthalmologist to appropriate, timely treatment of GCA.

I. Follow-up examination at 1 week A. Acuity: 20/60 O.U.; no relative afferent pupillary defect (RAPD); HRR 2/6 O.U.; visual fields: diffuse central loss O.U. B. Slitlamp exam 1. 2 + anterior chamber cells O.U. 2. No keratic precipitates, iris synechiae, or nodules C. Dilated fundus examination: 1+ anterior vitreous cells O.U.; bilateral disc edema with peripapillary serous exudates, bilateral macular exudates in star configuration D. Lymphadenopathy II. Diagnosis: Bilateral Optic Disc Edema With Macular Star All subsequent infectious and inflammatory workup negative, so presumed idiopathic. III. Terminology and Classification A. History 1. Initially described by Leber1 in 1916 a. Unilateral case of visual loss, optic disc edema, and macular exudates b. Believed primary pathology to be retinal c. Coined term stellate retinopathy 2. In 1977, Gass2 used fluorescein angiography to demonstrate that leakage came from optic disc capillaries, not retinal vasculature. a. Condition not a primary retinitis, but an optic neuropathy b. Coined term neuroretinitis

References
1. Weyand CM, Goronzy JJ. Giant-cell arteritis and polymyalgia rheumatica. Ann Intern Med. 2003;139(6):505-15. 2. Miller NR. Visual manifestations of temporal arteritis. Rheum Dis Clin North Am. 2001;27(4):781-97, vi.

2011 Subspecialty Day

Neuro-Ophthalmology

Section I: What Is Wrong With This Fundus? 7. Inflammatory bowel disease (IBD)

17

B. Classification by Brazis and Lee3-5 1. Idiopathic optic disc edema with macular star a. Primary optic neuropathy with secondary retinal involvement 2. Neuroretinitis a. Primary retinitis with associated optic disc edema b. Infectious etiology more likely to be identified C. Proposed classification scheme by Purvin et al6 1. Idiopathic neuroretinitis (NR) 2. Cat-scratch disease neuroretinitis (CSD NR) a. Bartonella henselae 3. Recurrent idiopathic neuroretinitis a. May be autoimmune b. ? Manifestation of ocular histoplasmosis syndrome IV. Etiologies5,6 A. Infectious causes 1. Viral: Herpes simplex, herpes zoster, hepatitis B, varicella, Epstein-Barr, influenza A, mumps, rubella, Coxsackie B, cytomegalovirus, chikungunya, dengue fever 2. Bacterial: cat-scratch disease (Bartonella species: henselae, quintana, grahamii, elizabethae), Rocky Mountain spotted fever, tuberculosis, Salmonella 3. Spirochetal: syphilis, Lyme, leptospirosis 4. Fungi: histoplasmosis, coccidiodomycosis, actinomycosis 5. Protozoan: toxoplasmosis 6. Nematodal: toxocariasis (diffuse unilateral subacute neuroretinitis [DUSN]) 7. Postvaccination: rabies B. Noninfectious causes 1. Vascular a. Hypertension: May have hemorrhages, cotton-wool spots, exudates, vascular tortuosity b. Diabetic papillopathy c. Retinal vein occlusion (central or branch) d. IRVAN syndrome (idiopathic retinal vasculitis and neuroretinitis) 2. Papilledema/increased intracranial pressure 3. Posterior vitreous traction 4. Optic disc/juxtapapillary tumors 5. Sarcoidosis 6. Polyarteritis nodosum

8. TINU syndrome (tubulointerstitial nephritis and uveitis) 9. Note: Multiple sclerosis is not associated with bilateral optic disc edema with macular star.7 V. Clinical Presentation of Optic Disc Edema with Macular Star4-6 A. Demographics 1. Usually affects young adults, though age range is broad (4-64) 2. No sex predilection B. Bilateral in up to 33% of cases In presumed unilateral cases, contralateral eye may have funduscopic findings but be asymptomatic. C. Pain may be present. D. 50% may have viral prodrome. E. Visual acuity ranges from 20/15 to no light perception. F. Dyschromatopsia (may be severe) G. Visual fields 1. Central or cecocentral scotoma most common 2. Altitudinal or arcuate defects are possible. 3. Less likely generalized constriction H. RAPD: May be present if unilateral or asymmetric I. Optic disc edema: May be diffuse or sectoral 1. Peripapillary hemorrhages 2. Peripapillary serous detachment J. Macular involvement: May take 1-2 weeks to develop K. Vitreous cells present in up to 90% of cases VI. Diagnostic Workup5,6 A. If noninfectious etiology suspected: 1. Check blood pressure 2. Serum glucose, hemoglobin A1C 3. Consider neuroimaging; lumbar puncture with opening pressure, cerebrospinal fluid contents B. If infectious etiology suspected: 1. Serologic testing: Bartonella IgM, IgG titers, Lyme titers, rapid plasma reagin (RPR), FTAAbs, toxoplasmosis, toxocariasis 2. Purified protein derivative (PPD) 3. Chest radiograph (CXR) C. Ancillary testing 1. OCT may show peripapillary serous retinal detachment prior to development of macular exudates.

18

Section I: What Is Wrong With This Fundus? 2. Fluorescein angiography (FA) a. Identify disc leakage and peripapillary dye staining b. May show other eye involvement in presumed unilateral cases 3. Fundus autofluorescence to help visualize macular exudates 4. MRI of brain/orbits a. May show enhancement of optic disc, retrobulbar optic nerve, and/or nerve sheath b. Should not show white matter demyelinating lesions as in MS7 VII. Management6 A. Prognosis 1. Typically very good, with recovery of visual function as optic disc edema and macular exudates resolve 2. Rarely in cases of recurrent idiopathic neuroretinitis, visual function may be impaired long term. B. Treatment 1. If infectious cause suspected (ie, cat scratch disease): may consider antimicrobial therapy a. Adults: ciprofloxacin or azithromycin b. Children: azithromycin or sulfamethoxazoletrimethoprim 2. If noninfectious cause suspected, may consider high-dose oral corticosteroids 3. If recurrent neuroretinitis: a. High-dose intravenous or oral corticosteroids b. Slow taper to prednisone 10 mg every other day c. Long-term immunosuppression may be required (ie, azathioprine). VIII. Neuro-Ophthalmic Red Flags in Case Presentation Initially, it seemed patient may have had increased intracranial pressure (ICP) from idiopathic intracranial hypertension (young, morbidly obese female with history of headaches), but there were several red flags suggesting an alternate diagnosis: A. Lack of symptoms of increased ICP (ie, tinnitus, TVOs, diplopia) B. History of viral prodrome C. Initial exam revealed retinochoroidal infiltrates (prior to macular star formation) D. After initial workup was negative, exam at 1 week revealed anterior chamber reaction with mild vitritis, and now fully developed macular star formation.

2011 Subspecialty Day

Neuro-Ophthalmology

References
1. Leber T. Die pseudonephritischen Netzhauterkrankungen, die Retinitis stellata: Die Purtschersche Netzhautaffektion nack schwerer Schadelverletzung. In: Graefe AC, Saemische T, eds. Graefe-Saemisch Handbuch der Augerheilkunde, 2nd ed. Leipzig, Germany: Engelmann, 1916:1319. 2. Gass JD. Diseases of the optic nerve that may simulate macular disease. Trans Sect Ophthalmol Am Acad Ophthalmol Otolaryngol. 1977; 83(5):763-770. 3. Dreyer RF, Hopen G, Gass JD, Smith JL. Lebers idiopathic stellate neuroretinitis. Arch Ophthalmol. 1984; 102(8):1140-1145. 4. Maitland CG, Miller NR. Neuroretinitis. Arch Ophthalmol. 1984; 102:1146-1150. 5. Brazis PW, Lee AG. Optic disk edema with a macular star. Mayo Clin Proc. 1996; 71(12):1162-1166. 6. Purvin V, Sundaram S, Kawasaki A. Neuroretinitis: review of the literature and new observations. J Neuroophthalmol. 2011; 31(1):58-68. 7. Parmley VC, Schiffman JS, Maitland CG, Miller NR, Dreyer RF, Hoyt WF. Does neuroretinitis rule out multiple sclerosis? Arch Neurol. 1987; 44:1045-1048.

Case #3: Unilateral Optic Disc Edema


Byron L Lam MD
Answer
Orbital MRI, complete blood count, rapid plasma reagin, fluorescent treponemal antibody, angiotensin converting enzyme, and Lyme sigma panel, as well as optic nerve ultrasound, were all normal or negative. No change in optic disc appearance occurred with 3 years of followup. Final diagnosis: Anomalous optic disc O.S. with pseudoedema.

Teaching Points
1. Unilateral pseudoedema may be caused by a variety of conditions, including anomalous optic disc and unilateral optic disc drusen. 2. Unilateral pseudoedema from anomalous optic disc is a diagnosis of exclusion. Features of anomalous optic disc causing pseudoedema typically include (a) small optic cup and small disc, (b) no hemorrhage, (c) ruddy optic disc appearance with no arteriolar dilation. 3. True unilateral disc edema with normal visual function may occur from a variety of conditions, including compressive (eg, meningioma, thyroid eye disease), vascular (eg, impending anterior ischemic optic neuropathy), and inflammatory (preLeber hereditary optic neuropathy disc edema).

2011 Subspecialty Day

Neuro-Ophthalmology

Section I: What Is Wrong With This Fundus? C. Seen in small crowded discs with anomalous branching patterns

19

Case #4: Visual Field Loss With an Afferent Pupillary Defect


Fiona E Costello MD
Answer
Diagnosis Based on visual field results, retinal inflammatory process such as an acute idiopathic blink spot enlargement spot (AIBSE) syndrome, multiple evanescent white dot syndrome (MEWDS) or acute zonal occult outer retinopathy (AOOR) should be considered. Multifocal ERG in the left eye demonstrating enlarged blind spot. See Figure 1. Diagnosis: Acute idiopathic blind spot enlargement syndrome

D. Scleral canal may be narrow and lead to intracellular mitochondrial calcification. E. Rupture of axons provides extracellular calcium and nidus for further calcium deposition. F. Develop in childhood and progress through adulthood G. Visible drusen in children are rare until second decade of life. H. More common in females, and at least 2/3 of cases are bilateral. II. Symptoms A. Often asymptomatic and found on routine exam B. May have transient visual obscurations C. Arcuate visual field defects are common. D. May have a higher incidence of migraine E. Acute visual loss rare but may cause peripapillary SRN, AION, CRVO. III. Clinical Findings A. Visual field loss common 1. More common in eyes with visible, as opposed to buried, drusen 2. Progressive field loss possible at rate of 1%-2% per year. 3. Retinal NFL thinning on OCT common B. Disc appearance 1. Disc may appear normal, mildly elevated, or as pseudopapilledema. 2. Usually small and crowded with absent cup 3. Hemorrhages, exudates, Paton lines not seen 4. Retinal vascular branching pattern anomalous with extra vessels 5. Disc appearance often changes over time as elevated discs in childhood show visible drusen in adulthood with anterior migration of the drusen. IV. Diagnosis A. B-scan echography shows highly reflective rounded shadows. B. Fundus autofluorescence is more sensitive than ophthalmoscopy. C. OCT shows retinal NFL thinning. V. Clinical Course A. 1%-2% rate of NFL loss per year B. Nonarteritic anterior ischemic optic neuropathy a high risk C. CRAO, CRVO, peripapillary subretinal neovascularization possible

Figure 1.

Teaching Points
Atypical findings for optic neuritis No eye pain Prominent photopsias Normal color vision Visual field not consistent with optic neuropathy but also not functional in appearance.

Remember: While 30% of optic neuritis patients have phosphenes, always keep the outer retinal disorders in mind.

Case #5: Bilateral Disc Edema


R Michael Siatkowski MD
Answer and Teaching Points
I. Pathophysiology A. Basophilic calcified bodies containing mucopolysaccharides, amino acids, DNA, RNA, iron B. Likely due to alteration of axoplasmic transport and axonal degeneration of ON fibers

20

Section I: What Is Wrong With This Fundus? VI. Treatments No treatment has proven beneficial to stop progression of visual field loss. A. Lowering IOP to increase perfusion pressure tried 1. SLT, ALT, topical agents with neuroprotective capabilities may be used. 2. Surgical trabeculectomy used in some severe cases B. Optic nerve sheath fenestration has been tried but no good evidence of efficacy. C. Surgical removal of large drusen reported but high risk of visual loss.

2011 Subspecialty Day

Neuro-Ophthalmology

Selected Readings
1. Lam BL, et al. Drusen of the optic disc. Curr Neur Neurosci Rep. 2008; 8:404-408. 2. Lee AG, Zimmerman MB. The rate of visual field loss in optic nerve head drusen. Am J Ophthalmol. 2005; 139:1062-1066. 3. Heidary G, Rizzo JF. Use of optical coherence tomography to evaluate papilledema and pseudopapilledema. Semin Ophthalmol. 2010; 25:198-205. 4. Spencer TS, et al. Progression from anomalous optic discs to visible optic disc drusen. J Neuroophthalmol. 2004; 24:297-298.

right middle cerebral peduncle. The azathioprine appears to have contributed to clinical and radiographic remission, with subsequent MRI finally showing stabilization of white matter and optic nerve lesions. To date he has not developed myelopathy or spinal lesions. During his clinical course, skin biopsies on two occasions were consistent with mild vasculitis by hematoxylin and eosin staining as well as immunoflourescent methods (perivascular deposition of IgM and C1q) previously described in association of cases of autoimmune or collagen-vascular-disease associated optic neuropathy.1 Evaluation for sarcoid,2,3 including full body gallium, chest CT, and SPECT scans, revealed only minimal low-grade parotid gallium uptake. When immunohistochemical analysis for aquaporin 4 in neuromyelitis optica (NMO)4 became available, initial tests were negative. However, co-immunoprecipitation studies, which were not validated at the time they were performed, were positive and described as the highest values Mayo clinic had obtained. The current commercially available ELISA studies for NMO have not been drawn because of financial considerations. Evaluation during the patients decade of care showed normal: repeated lumbar punctures, CSF multiple sclerosis panel (oligoclonal bands, IgG synthesis rate, and index, MBP), CSF VDRL, ACE, and cytology, and serum ACE, ANA, dsDNA, FTA-ABS, RF, HIV, heavy metal, and tox screen (mildly elevated arsenic), ANCA, urine analysis, anticardiolipin, and hypercoaguable profile. The patient declined optic nerve and brain biopsy on multiple occasions.

Case #6: Optic Neuritis... or Not?


Roger E Turbin MD
Answer
Discussion: Additional Clinical Course Unfortunately, with each attempted corticosteroid taper (Months 1, 8, 11, 19, 25, and 27) 7 ipsilateral episodes of loss of vision in the right eye ensued. With intravenous corticosteroid re-treatment the ipsilateral visual acuity, which had dropped to count fingers range, initially returned to normal. The acuity in the later episodes only returned to 20/200. A variety of corticosteroidsparing, cytotoxic, immunomodulatory, and biologic agents failed to stabilize his clinical course (methotrexate, mycophenylate, azathioprine, standard-dose cyclophosphamide, high-dose beta interferon, natalizumab, plasmapharesis, and IVIG). At Month 33, he lost vision in the left eye for the first time to no light perception, which returned to 20/200 with myeloablative (600 mg/meter2 to 4.5 gram) induction dose cyclophosphamide. Following a cycle of growth stimulating factor (filgrastim), visual acuity dropped to no light perception bilaterally. His clinical course was further complicated by corticosteroid-induced psychosis as well as DVT with main stem pulmonary arterial embolism. To date, 9 years later, he remains no light perception bilaterally, maintained on azathioprine to prevent the development of spinal lesions as a case of atypical Devics disease (NMO). Interestingly, he has experienced only mild temporary sensory loss in his right face despite the development of numerous scattered white matter abnormalities and enhancing lesions, including his

Teaching Points
1. Multiple recurrence, chronic progression, or corticosteroid dependence should prompt evaluation for diseases other than demeylinating optic neuritis. Atypical characteristics of optic neuritis in adults also include absence of pain, lack of significant visual improvement, age over 40,5 as well as atypical clinical course, prominent optic nerve swelling or hemorrhages, and retinal exudates.6 2. As in this case, it may be difficult to distinguish optic neuritis from other mimics at first presentation, and clinical follow-up is warranted to sort out atypical cases. Fortunately the incidence of idiopathic demyelinating optic neuritis (associated with multiple sclerosis) far exceeds other rare forms of inflammatory, autoimmune, neoplastic, and infiltrative optic neuropathies.

References
1. Kupersmith MJ, Burde RM, Warren FA, et al. Autoimmune optic neuropathy: evaluation and treatment. J Neurol Neurosurg Psychiatry. 1988; 51:1381-1386. 2. Frohman LP, Guirgis M, Turbin RE, et al. Sarcoidosis of the anterior visual pathway: 24 new cases. J Neuroophthalmol. 2003; 23:190197. 3. Koczman JJ, Rouleau J, Gaunt M, et al. Neuro-ophthalmic sarcoidosis: the University of Iowa experience. Semin Ophthalmol. 2008; 23:157-168. 4. Pezold A, Pittock S, Maggiore C, et al. Neuromyelitis optica-IgG (aquaporin-4) autoantibodies in immune mediated optic neuritis. JNeurol Neurosurg Psychiatry. 2010; 81:109-111.

2011 Subspecialty Day

Neuro-Ophthalmology

Section I: What Is Wrong With This Fundus?

21

5. Lee AG, Lin DJ, Kaufman M, et al. Atypical features prompting neuroimaging in acute optic neuropathy in adults. Can J Ophthalmol. 2000; 35:325-330. 6. Balcer LJ. Optic neuritis. N Engl J Med. 2006; 354:1273-1280.

Case #7: Optic Atrophy Whats Next?


Dean M Cestari MD
Answer: Optic atrophy with optociliary shunt vessel from optic nerve sheath meningioma Teaching Points
Introduction Optic neuropathies can result from ischemic, inflammatory, hereditary, toxic, compressive, or infiltrative etiologies that can vary from benign to life threatening. Rarely can the etiology of an optic neuropathy be determined on the basis of a single clinical finding, and there is considerable overlap between the clinical features of the relatively common causes and the potentially life-threatening etiologies of an optic neuropathy. The signs and symptoms of an optic neuropathy are reviewed below.1 Signs of an Optic Neuropathy Central (Snellen) acuity Patients with an optic neuropathy can have visual acuity (VA) that ranges from normal to no light perception. The degree of reduction in VA does not help to differentiate the cause of an optic neuropathy, including from compressive lesions such as tumors or aneurysms. Compressive lesions can reduce VA if the more central fibers within the optic nerve are compressed, but less significant compression may not decrease VA if only the nasal (crossing) or temporal (uncrossed) outflow from the retina is affected.2 Optic nerve compression, therefore, produces a wide range of central acuity. Color vision Dyschromatopsia is a very sensitive indicator of optic nerve dysfunction, and asymmetry of color vision is an important clue to the presence of an optic neuropathy. A commonly used test is the Ishihara color plates, which, despite not providing quantitative information about the color defects, have a very high yield in revealing optic nerve dysfunction. A significant delay in recognizing the color plates in one eye, even if the interpretation of the number is correct, also suggests the presence of an optic neuropathy. Pupil A relative afferent pupillary defect (RAPD) is typically seen in the presence of a unilateral optic neuropathy, and rarely with severe macular disease. Fundus In the presence of an optic neuropathy, the funduscopic appearance of the optic nerve can vary from normal to swollen to pale. Compressive lesions of the optic nerve can cause chronic optic

disc swelling that may be associated with shunt vessels that represent collateral vessels from the retinal to ciliary circulation (see Table 1) and pseudodrusen of the optic nerve head, almost always in the absence of hemorrhages. Compressive lesions may also cause increased cupping of the optic disc.3 This cupping should be easily distinguished from that associated with glaucoma because the neuroretinal rim becomes pale with compressive lesions, whereas it retains its normal pink appearance in glaucoma.4 Optic nerve pallor may develop with any lesion of the afferent pathway that is presynaptic to the lateral geniculate nucleus, and this pallor may be diffuse or sectoral. Any presynaptic injury may cause diffuse, altitudinal, sectoral, or temporal atrophy. Table 1. Differential Diagnosis of Optociliary Shunt Vessels
Central retinal vein occlusion Glaucoma Optic nerve mass (meningioma, glioma) Idiopathic intracranial hypertension Chronic papilledema

Visual fields Visual field defects that reflect involvement of the optic nerve include central/paracentral and arcuate scotomas, altitudinal defects, nasal steps, temporal wedges, and the less specific finding of generalized depression. No visual field defect is specific for a specific etiology. Tempo of vision loss in an optic neuropathy Patients with vascular, inflammatory, or demyelinating optic neuropathies typically present with acute to subacute vision loss over a period of hours to days. Compressive optic neuropathies typically cause visual symptoms that are initially very subtle and vague and they progress over weeks to months and in some cases years. Optic Nerve Sheath Meningiomas Optic nerve sheath meningiomas (ONSMs) are rare, benign tumors of the optic nerve; 60%-70% of cases occur in middleaged females. Rarely, they can be seen in children. The tumors grow from the dural sheath that surrounds the optic nerve, and as the tumor grows, it slowly compresses the optic nerve. This causes slow loss of vision in the affected eye over a period of months to years. Rarely, it may affect both eyes at the same time. In some patients, the tumor grows so slowly that treatment is not necessary. Standard treatments are observation, surgery, radiation therapy, and combinations of the above.5 Incidence About 1%-2% of all meningiomas are optic nerve sheath meningiomas. Meningiomas have an incidence of ~4.18/100,000 persons each year. Thus, ~10,000 meningiomas are diagnosed in the United States each year, corresponding to ~100 cases of ONSM each year in the United States. There is an increased incidence in patients with neurofibromatosis type II. Symptoms The most common symptom of ONSM is a gradual loss of vision in one eye. In a minority of patients this may be intermittent, at

22

Section I: What Is Wrong With This Fundus?

2011 Subspecialty Day

Neuro-Ophthalmology

least to begin with. Less common symptoms include pain in the affected eye, protrusion of the eye, or double vision. Diagnosis Clinical examination will show an abnormal optic disc, either swollen or atrophic. Optociliary shunt vessels may be seen. Visual acuity is usually but not always reduced. When ONSM is suspected, MRI of the brain and orbits with gadolinium should be performed. Treatment Reports of patients with ONSM having no change in their vision for multiple years are not uncommon. Most ophthalmologists will observe patients unless their VA becomes worse than 20/40. If loss of vision occurs, radiation therapy is generally indicated. Surgery has no role in the management of ONSM.6

References
1. Cestari DM, Rizzo JF III. The neuroophthalmic manifestations and treatment options of unruptured intracranial aneurysms. Int Ophthalmol Clin. 2004; 44(1):169-187. 2. Frisen L. The neurology of visual acuity. Brain. 1980; 103(3):639670. 3. Bianchi-Marzoli S, Rizzo JF III, Brancato R, Lessell S. Quantitative analysis of optic disc cupping in compressive optic neuropathy. Ophthalmology 1995; 102(3):436-440. 4. Trobe JD, Glaser JS, Cassady J, Herschler J, Anderson DR. Nonglaucomatous excavation of the optic disc. Arch Ophthalmol. 1980; 98(6):1046-1050. 5. Dutton JJ. Optic nerve sheath meningiomas. Surv Ophthalmol. 1992; 37(3):167-183. 6. Turbin RE, Thompson CR, Kennerdell JS, Cockerham KP, Kupersmith MJ. A long-term visual outcome comparison in patients with optic nerve sheath meningioma managed with observation, surgery, radiotherapy, or surgery and radiotherapy. Ophthalmology 2002; 109(5):890-899; discussion 899-900.

2011 Subspecialty Day

Neuro-Ophthalmology

Section II: What Is Wrong With This Eyelid/Pupil?

23

Section II: What Is Wrong With This Eyelid/Pupil? Answer and Teaching Points
Case #8: A Dilated Pupil With Blurred Vision
Franois-Xavier Borruat MD
Answer and Teaching Points
Introduction The sudden onset of anisocoria, especially occurring after trauma or when accompanied by headaches, is always a challenge for the clinician. An accurate diagnosis is mandatory to distinguish the patient with a real emergency (ie, posterior communicating artery aneurysm, subdural hematoma, internal carotid artery dissection) from other, more benign situations. The differential diagnoses of anisocoria can be summarized as follows: essential or physiological anisocoria ophthalmological anisocoria neurological anisocoria pharmacological anisocoria agent competes with acetylcholine at the receptor site, the use of a parasympathomimetic agent such as pilocarpine will not be able to induce miosis. Mydriasis will progressively subside over hours or days, depending on the offending substance. Apart from eyedrops such as tropicamide, cyclopentolate, scopolamine, and atropine, patients eyes can become contaminated from contact with several plants and flowers, such as deadly nightshade (Atropa belladonna), henbane (Hyoscamus niger), jimson weed (Datura stramonium), angels trumpet (Datura suaveolens), and blue nightshade (Solanum dulcamara). A classic but sometimes overlooked cause of unilateral fixed mydriasis is the use of cutaneous patch of scopolamine to treat car sickness: at the end of the trip, the patient removes the patch with one hand, forgets to wash the hands, and rubs one eye. Adrenergic agent The mydriasis induced by the use of epinephrine or phenylephrine (both direct sympathicomimetic) can be quite large, depending on the concentration used and on the color of the iris. However, because the parasympathetic receptors of the iris sphincter muscle are not pharmacologically blocked (the iris sphincter muscle is stronger than the iris dilator muscle), illumination will result in a certain degree of miosis, and the use of pilocarpine will result in a significant miosis. Topical cocaine, an indirect sympathicomimetic agent, has similar effects on the pupil. Pharmacological Miosis Miosis can result from either a parasympathicomimetic or a sympathicolytic effect. Parasympathicomimetic agent Topical drug-induced miosis results mostly from instillation of pilocarpine, an acetylcholine-like substance that stimulates directly the cholinergic receptors. Miosis is pronounced, and the pupil cannot be dilated by the use of either sympathicomimetic or parasympathicolytic drops. This situation is rare in clinical practice. Brimonidine Brimonidine is used in the treatment of glaucoma because its 2-receptor agonist action induces vasoconstriction and reduces the secretion of aqueous humor. However, brimonidine appears to reduce sympathetic function at the iris dilator muscle, presumably by stimulating presynaptic inhibitory 2 receptors. A slight superior eyelid ptosis and miosis will result. Usually brimonidine is prescribed for the treatment of glaucoma in both eyes, and no interocular asymmetry will be noticed. However, in the case of unilateral treatment, brimonidine will induce an ipsilateral pseudo-Horner syndrome. It will behave as a true Horner syndrome when challenged with either topical cocaine drops or topical apraclonidine drops. The syndrome is reversible after cessation of brimonidine. It is important to keep in mind this possibility, in order to avoid submitting the patient to useless and potentially harmful investigations.

History and slitlamp examination are necessary to rule out/ confirm the possibility of an ophthalmological cause for the anisocoria (previous eye trauma or surgery, uveitis, congenital iris anomalies...). Once an ophthalmological cause is excluded, examination of the pupils under different lighting conditions (darkness vs. bright environment) and during convergence is necessary for the clinician to decide which type of anisocoria is present: essential or physiological, neurological, or pharmacological anisocoria. Pharmacological Cause of Anisocoria The size of the pupil depends on the interaction of two different muscles, the iris dilator muscle and the iris sphincter muscle, which are both innervated by parasympathetic (cholinergic receptors) and sympathetic (adrenergic receptors) nerve fibers. Cholinergic stimulation results in iris sphincter muscle contraction (miosis), and adrenergic stimulation induces mydriasis (iris dilator muscle contraction). Additionally, both parasympathetic and sympathetic fibers inhibit the antagonistic muscle to produce a stronger effect on the pupil size. Sympathetic fibers exhibit three types of receptors: 1, 2, 1. Activation of 2 receptors stimulates the iris dilator muscle (mydriasis), while 1 stimulation will induce miosis because it will contract the iris sphincter muscle. Pharmacologically, the pupil size can therefore be influenced by either stimulating or inhibiting either the cholinergic or the adrenergic receptors. Pharmacological Mydriasis This can result from the use of either an anticholinergic or an adrenergic agent. Anticholinergic agent This is the most frequent situation of a drug-induced mydriasis. The pupil is large and completely nonreactive to light and convergence. Slitlamp examination is unremarkable. Because the

24

Section II: What Is Wrong With This Eyelid/Pupil?

2011 Subspecialty Day

Neuro-Ophthalmology

Take Home Messages Unilateral mydriasis: A large dilated pupil that does not react to 2% pilocarpine is due either to a previous trauma/ lesion to the iris or to a pharmacological blockade. It never results from a neurological cause. Horner syndrome in a patient treated for glaucoma: Examine the possibility that brimonidine is used in only one eye.

History of allergies Contact lens wear Look for Pertinent Clinical Features Ptosis, ipsilateral mydriasis, and paresis of some or all of the extraocular ocular muscles innervated by the oculomotor nerve Ptosis in the setting of miosis and anhidrosis Anisocoria greater in dim light, with the affected pupil being smaller Ptosis and/or lid retraction with a larger pupil Structural changes in the pupillary sphincter due to trauma Tonic pupil Dilated pupil of dorsal midbrain syndrome Ptosis that fatigues Ptosis and ocular duction deficits in one or both eyes Ptosis and/or lid retraction and ocular surface disease Lid retraction associated with eyelid lag and lagophthalmos Lid retraction and foreshortened upper eyelid Lid retraction and vertical deviation Apraxia of eyelid opening Describe Appropriate Testing and Evaluation to Establish the Diagnosis in the Office Setting Measure the pupil in both dark and light environments. Slitlamp biomicroscopy to determine the presence of local structural factors as possible causes of anisocoria Pharmacologic testing * Anisocoria greater in dim illumination * Anisocoria greater in bright illumination Slitlamp biomicroscopy for ocular surface disease such as conjunctival injection, papillary or follicular reaction Check the ocular motility and alignment Lower eyelid position relative to inferior limbus and contralateral side Margin-reflex distance measures (MRD1, MRD2) Vertical interpalpebral fissure height measurements Upper eyelid crease measurements Levator muscle function measurements Reversal of ptosis on downgaze Phenylephrine testing Check for enhanced ptosis Check for Cogan lid twitch sign Ice pack testing Tensilon testing Visual field testing Review of old photographs may be helpful. Differential Diagnosis Horner syndrome Cranial nerve (CN) III palsy Congenital myogenic ptosis Acquired myogenic ptosis (ie, myasthenia gravis, chronic progressive external ophthalmoplegia, etc. ) Aponeurotic ptosis Mechanical ptosis Traumatic ptosis Blepharospasm, hemifacial spasm, or facial nerve misdirection Pseudoptosis

References
1. Kardon RH. Anatomy and physiology of the pupil. In: Walsh & Hoyts Clinical Neuro-Ophthalmology, 5th ed. Philadelphia: Williams and Wilkins, 1998: 847-898. 2. Digre KB. Principles and techniques of examination of the pupils, accommodation, and the lacrimal system. In: Walsh & Hoyts Clinical Neuro-Ophthalmology, 5th ed. Philadelphia: Williams and Wilkins, 1998: 999-1005. 3. Antonio-Santos AA, Santo RN, Eggenberger ER. Pharmacological testing of anisocoria. Expert Opin Pharmacother. 2005; 12:20072013. 4. Borruat F-X, Kawasaki A. Horner-like syndrome due to brimonidine 0.2%. Poster presented at the Annual Meeting of the American Academy of Ophthalmology; 2003; Chicago. 5. Kesler A, Shemesh G, Rothkoff L, Lazar M. Effect of brimonidine tartrate 0.2% ophthalmic solution on pupil size. J Cataract Refract Surg.2004; 8:1707-1710.

Case #9: Which Eyelid Is Abnormal?


Marie D Acierno MD
Answer and Teaching Points
Introduction It is the role of the clinician to gather pertinent information in the patient history and to analyze the accompanying clinical features to determine which eye or eyelid is the cause of the patients symptoms. As the lecture will demonstrate, either the higher or lower lid may be pathologic. This outline will serve as a preliminary guide to the clinician in the office setting to help to identify the abnormal eyelid. The discussion will elaborate on these details and refine the clinicians examination and decision-making process. Pertinent Elements of the History Duration of eyelid position findings Fluctuations in eyelid position during the day or with fatigue Early morning eyelid swelling (dependent orbital edema) Diplopia Proptosis Orbital discomfort, pain Changes in pupillary size Presence of painneck pain, shoulder pain, or headache especially in the setting of trauma Eyelid erythema Transient, insidious, or rapid visual loss History of trauma or previous ocular surgery

2011 Subspecialty Day

Neuro-Ophthalmology

Section II: What Is Wrong With This Eyelid/Pupil?

25

Previous globe/eyelid trauma (with or without iris involvement) Thyroid eye disease Lid retraction from sympathetic hyperfunction (ie, ClaudeBernard syndrome) Lid retraction of midbrain disease (ie, Collier sign of dorsal midbrain syndrome) Lid retraction post strabismus surgery (ie, vertical recti muscle recession) Lid retraction associated with aberrant regeneration Blepharoplasty with excessive skin excision Pseudo-lid-retraction Patient Management in Terms of Treatment and Follow-up Based on etiology Imaging, when appropriate: Urgent for patients with suspected aneurysmal third nerve palsy, carotid dissection with an associated Horner syndrome, and compressive optic neuropathy with visual loss secondary to thyroid orbitopathy Thyroid function testing, when appropriate Neuromuscular junction antibody testing; EMG testing, when appropriate Follow-up clinical examination to document stability or progression Report new neurologic symptoms Surgical procedures for ptosis and eyelid retraction Eyelid crutches for ptosis when myopathic (ie, CPEO) Botulinum toxin injections for lid retraction in limited cases

tumor was removed and radiated, the dorsal midbrain syndrome persisted. II. Dorsal midbrain syndrome (also known as Parinaud syndrome, pretectal syndrome, Sylvian aqueduct syndrome) is characterized by: A. Disordered supranuclear gaze, especially upgaze paresis Thought to be caused by involvement of the interstitial nucleus of Cajal (INC) and also the projections in the posterior commissure. Saccades are more involved than pursuit. The use of oculovestibular movements may be helpful to show that the upgaze deficit is supranuclear. B. Light-near dissociation of the pupils (pupils constrict better to near than to light) Thought to be due to involvement of the pupillary fibers in the pretectum with preservation of the accommodative fibers from the Edinger-Westfall nucleus. Early on, the pupil may even mimic a tonic pupil. Of all the findings in Keanes large review of pretectal syndrome, pupillary abnormalities were most common. C. Lid retraction usually in primary position (sometimes known as Collier sign) Eyelid findings may predominate in some cases. In primary gaze, look for lid elevation above the limbus. Unlike lid retraction in thyroid orbitopathy, the lid retraction is not generally seen in down gaze. The cause is thought to be either excessive superior rectus innervation or disinhibition of the central caudal subnucleus of cranial nerve III that leads to levator activation. The lid retraction seems to parallel the amount of elevation deficit: large elevation deficit, more lid retraction. D. Convergence retraction nystagmus There is co-contraction of the extraocular muscles; this causes both convergence and retraction of the globe and is thought to be due to the posterior commissure lesion. This nystagmus may be brought out by using an optokinetic nystagmus drum moving downward, forcing the upward saccade and the retraction nystagmus. E. Convergence spasm or convergence insufficiency F. Accommodative spasm and/or paresis G. Pseudo-abducens palsy The abducting eye may move slower than the adducting eye. H. Skew deviation (a vertical misalignment due to brainstem involvement) III. Symptoms The typical symptoms of this disorder are usually diplopia, blurred vision, especially at near, and trouble looking up. Other symptoms like headache, nausea, and vomiting would suggest increased intracranial pressure from obstructive hydrocephalus.

References
1. American Academy of Ophthalmology. Basic and Clinical Science Course, Section 7: Orbit, Eyelids, and Lacrimal System. San Francisco: AAO, 2009. 2. American Academy of Ophthalmology. Basic and Clinical Science Course. Section 5: Neuro-ophthalmology. San Francisco, 2009. 3. Trobe JD. Rapid Diagnosis in Ophthalmology Neuroophthalmology. China: Mosby Elsevier, 2008. 4. Miller NR and Newman NJ. Walsh & Hoyts Clinical NeuroOphthalmology. 6th ed. Baltimore: William & Wilkins, 2005: vol. 1, 1177-1204. 5. Glaser JS. Neuro-Ophthalmology. 3rd ed. Philadelphia: Lippincott Williams & Wilkins, 1999: 59-63; 493-502.

Case #10: The Opposite of Ptosis


Kathleen B Digre MD
Answer and Teaching Points
I. Diagnosis: Dorsal midbrain syndrome with lid retraction and light near dissociation This young man had a contrast enhanced MR scan that showed a pineal tumor compressing his dorsal midbrain. The tumor was a pineal germinoma. While the

26

Section II: What Is Wrong With This Eyelid/Pupil? IV. Localization Because the findings are so characteristic, as described above, localization is to the dorsal midbrain area. MR imaging will show most lesions, but if none is found, review the images yourself with the neuroradiologist. V. Causes Causes of this syndrome are several: A. Tumors, especially pineal region tumors including pineal cytomas or pinealblastomas (highly malignant), germinoma, cysts, and glioma B. Infarction from arteries off of the top of the basilar artery (rare as an isolated sign) C. Hemorrhages from hypertension or cavernomas in the brainstem D. Hydrocephalus (especially obstructive); shunt failure is an infrequent cause of dorsal midbrain syndrome E. Demyelinating disease such as MS F. Paraneoplastic syndromes G. Others include: encephalitis, tentorial herniation, AIDS, sarcoid of the mid-brain VI. Treatment Treatment is directed toward treatment of the underlying diagnosis. However, there are many bothersome problems to the patient to consider, including diplopia (which may be assisted by prisms or surgery). However, frequently the diplopia is complex and associated with nystagmus, making treatment more challenging. The accommodative paresis will make reading difficult; presbyopic spectacle correction may be helpful. VII. Key Points A. Consider the dorsal midbrain syndrome if you see someone with lid retraction and a light near dissociation. Further clinical testing may be required; look at vertical saccadesis there convergence retraction nystagmus? Use the optokinetic nystagmus drum in a downward movement to demonstrate the nystagmus. B. Always image anyone you suspect of a dorsal midbrain syndrome; there is almost always a cause and a lesion. C. If anyone with a known shunt has a dorsal midbrain syndrome, consider obstructive hydrocephalus and shunt failure as the cause.

2011 Subspecialty Day

Neuro-Ophthalmology

4. Saunders M, Guinane C, MacFarlane M, Tarr K, Bhatti MT, Pincus DW. A diplopia dilemma. Surv Ophthalmol. 2006; 51(1):68-74. 5. Lee AG, Brown DG, Diaz PJ. Dorsal midbrain syndrome due to mesencephalic hemorrhage: case report with serial imaging. J Neuroophthalmol. 1996; 16(4):281-285. 6. Chou SY, Digre KB. Neuro-ophthalmic complications of raised intracranial pressure, hydrocephalus, and shunt malfunction [review]. Neurosurg Clin N Am. 1999; 10(4):587-608. 7. Baloh RW, Furman JM, Yee RD. Dorsal midbrain syndrome: clinical and oculographic findings. Neurology 1985; 35(1):54-60.

Case #11: Small Pupil, Droopy Eyelid... Diagnosis?


Michael Vaphiades DO
Answer and Teaching Points
Diagnosis: Episodic pupillary dilation O.D. associated with migraine and mechanical ptosis O.S. Horner syndrome results from a lesion affecting the sympathetic supply to the eye. The clinical features on the affected side include 1-2 mm of miosis (from denervation of the sympathetically innervated pupillary dilator muscle), with greater anisocoria in dim illumination and a dilation lag; mild upper lid ptosis measuring 1-2 mm due to denervation of the sympathetically supplied superior tarsal muscle and denervation of the lower eyelid retractors; and anhidrosis if the lesion is proximal to the carotid bifurcation. Lesions distal to the carotid bifurcation do not affect the sympathetic innervation of facial sweat glands. Ptosis may occur in association with a smaller pupil for reasons other than a Horner syndrome. Anisocoria combined with ptosis of unrelated cause may produce a pseudo-Horner syndrome. Although there are many causes of ptosis, one of the most frequent is the levator dehiscence-disinsertion syndrome. In these patients the aponeurosis of the levator palpebrae is structurally abnormal, with areas of dehiscence and disinsertion from the tarsal plate of the upper eyelid. It is a frequent cause of ptosis in the elderly but can also be seen in younger patients with history of ocular allergy, lid edema, long-term contact lens use, or lid trauma. Typical clinical findings include (1) excellent levator function (>12 mm), (2) high to absent lid fold, and (3) bilateral tendency in the aged. The physician should also be aware that myasthenia gravis combined with physiologic anisocoria may also produce a pseudo-Horner syndrome. Initial Pharmacologic Testing for Horner Syndrome: Cocaine and Apraclonidine Tests To test for the presence of a Horner syndrome, topical cocaine drops are instilled in each eye. A 4%, 5%, or 10% solution can be used. Often the use of a 10% solution will ensure that even dark brown irides, which are more resistant to dilating drops than lighter colored irides, get a full mydriatic dose. No more than 2 drops should be used because of the risk of producing a corneal epithelial defect. After 40-60 minutes, the pupils should be dilated. Cocaine requires normal sympathetic function to cause dilation. Therefore, if a sympathetic lesion is present, the Horner pupil will dilate less than the normal pupil. An anisoco-

References
1. The Neuro-Ophthalmology Virtual Education website: http:// NOVEL.utah.edu. 2. Liu GT, Volpe NJ, Galletta SL. Neuro-ophthalmology Diagnosis and Management. Philadelphia: W.B. Saunders, 2001. 3. Keane JR. The pretectal syndrome. Neurology 1990; 40:684-690.

2011 Subspecialty Day

Neuro-Ophthalmology

Section II: What Is Wrong With This Eyelid/Pupil?

27

ria of at least 0.8 mm after the drops are instilled will confirm the presence of Horner syndrome. An alternative medication to cocaine testing in Horner syndrome is the use of apraclonidine. It is an ocular hypotensive agent that has a weak direct action on alpha-1 receptors and therefore minimal to no clinical effect on the pupils of normal eyes. Patients with Horner syndrome, however, have denervation supersensitivity of the alpha-1 receptors in the iris stroma of the affected eye. With instillation of 1% or 0.5% apraclonidine, there is a reversal of anisocoria (affected pupil dilates), and patients may also experience reversal of the ptosis on the affected side. Establishing the diagnosis of a Horner syndrome can occur as soon as 36 hours after of the onset of the oculosympathetic dysfunction. There are serious causes of both anisocoria and ptosis that may not be a Horner syndrome. A systematic approach, including the use of topical cocaine or apraclonidine, is essential when confronted with a patient with ipsilateral ptosis and miosis.

References
1. Anderson RL, Dixon RS. Aponeurotic ptosis surgery. Arch Ophthalmol. 1979; 97:1123-1128. 2. Bremner F, Smith S. Pupillographic findings in 39 consecutive cases of Harlequin syndrome. J Neuroophthalmol. 2008; 28:171-177. 3. Jones LT, Quickert MH, Wobig JL. The cure of ptosis by aponeurotic repair. Arch Ophthalmol. 1975; 93:629-634. 4. Freedman KA, Brown SM. Topical apraclonidine in the diagnosis of suspected Horners syndrome. J Neuroophthalmol. 2005; 25:83-85. 5. Lebas M,Seror J,Debroucker T. Positive apraclonidine test 36 hours after acute onset of Horner syndrome in dorsolateral pontomedullary stroke. J Neuroophthalmol.2010; 30:12-17. 6. Meador CK. The art and science of non-disease. N Engl J Med. 1965; 272:92-94. 7. Sutula FC, Dortzback RK. Involutional blepharoptosis: a histopathological study. Arch Ophthalmol. 1980; 98:2045-2049. 8. Thompson B, Corbett J, Kline L, Thompson S. Pseudo-Horners syndrome. Arch Neurol. 1982; 39:108-111. 9. Thompson HS, Kardon R. Clinical importance of pupillary inequality. Focal Points (San Francisco: American Academy of Ophthalmology). 1992:10.

as well. No IOP-lowering agents were used subsequently, and the IOP never rose above 24 ever again. Subsequent postoperative visits all noted that the pupil was 6 mm in size, without comment on reactivity to light. Meanwhile the patient was pleased that the vision was markedly improved compared to preoperatively. Urrets-Zavalia first described the phenomenon of a fixed and dilated pupil following penetrating keratoplasty for keratoconus in 1963, and the condition was subsequently named UrretsZavalia syndrome. At the time and in subsequent reports, the feeling was that mydriatic agents, in combination with the nature of the surgery itself, resulted in tight peripheral lens-cornea apposition and resultant iris ischemia and/or peripheral synechiae. As a result, many of these patients had not only a dilated pupil, but raised IOP as well. Tuft and Buckley reported a case in 1995 in which the concept of iris ischemia was specifically implicated in combination with high IOP. The apparent contribution of keratoconus as a risk factor also led to speculation that the decreased corneal rigidity played a role in the final outcome. Davies and Ruben noted in 1975 that lack of a peripheral iridotomy seemed to be predictive of abnormal pupils after keratoplasty for keratoconus, suggesting a role for pupillary block. In recent years, deep lamellar keratoplasty for keratoconus was also noted to be associated with dilated pupil postoperatively. However, the condition does not appear to be restricted to only corneal transplant, nor to patients with prior diagnosis of keratoconus. Urrets-Zavalia syndrome has been noted after deep anterior lamellar keratoplasty, trabeculectomy, laser trabeculoplasty, and Descemet-stripping endothelial keratoplasty, as well as many other anterior segment procedures. In those cases reported in the literature, as well as this patient, pupillary block is the key mechanism, resulting in an atonic pupil due to high IOP and iris ischemia. Further examination of this patient reveals iris atrophy via transillumination techniques. If iris atrophy does not develop, these patients may show a partial reversal or recovery of pupil/iris function. Without a careful iris examination and record review, this patient could have been a fresh Adies pupil, having not yet developed light-near dissociation of the pupil nor any supersensitivity to cholinergic agents.

Selected Readings
1. Urrets-Zavalia A. Fixed dilated pupil, iris atrophy and secondary glaucoma: a distinct clinical entity following penetrating keratoplasty for keratoconus. Am J Ophthalmol. 1963; 56:257-265. 2. Tuft SJ, Buckley RJ. Iris ischaemia following penetrating keratoplasty for keratoconus (Urrets-Zavalia syndrome). Cornea 1995; 14(6):618-622. 3. Davies PD, Ruben M. The paretic pupil: its incidence and aetiology after keratoplasty for keratoconus. Br J Ophthalmol. 1975; 59:223-228. 4. Maurino V, Allan BD, Stevens JD, Tuft SJ. Fixed dilated pupil (Urrets-Zavalia syndrome) after air/gas injection after deep lamellar keratoplasty for keratoconus. Am J Ophthalmol. 2002; 133(2):266268. 5. Minasian M, Ayliffe W. Fixed dilated pupil following deep lamellar keratoplasty (Urrets-Zavalia syndrome). Br J Ophthalmol. 2002; 86(1):115-116. 6. Niknam S, Rajabi MT. Fixed dilated pupil (Urrets-Zavalia syndrome) after deep anterior lamellar keratoplasty. Cornea 2009; 28(10):1187-1190.

Case #12: Large Pupil That Reacts (A Bit)


Reid Longmuir MD
Answer and Teaching Points
Subsequent record review shows that after the endothelial transplant in the left eye, the IOP was 65 at a 6-hour postoperative visit. The patient was nauseated but had always assumed this was a reaction to anesthetic. The pupil was 6 mm at that visit, having been 4 mm in the dark preoperatively. The patient was noted to be in pupillary block at that visit, due to the air bubble used to support the endothelial graft in position. The pressure was reduced to 20 with anterior chamber paracentesis and air removal, and the pupillary block was broken with this approach

28

Section II: What Is Wrong With This Eyelid/Pupil?

2011 Subspecialty Day

Neuro-Ophthalmology

7. Jain R, Assi A, Murdoch IE. Urrets-Zavalia syndrome following trabeculectomy. Br J Ophthalmol. 2000; 84(3):338-339. 8. Espana EM, Ioannidis A, Tello C, Liebmann JM, Foster P, Ritch R. Urrets-Zavalia syndrome as a complication of argon laser peripheral iridoplasty. Br J Ophthalmol. 2007; 91(4):427-429. 9. Fourni P, Ponchel C, Malecaze F, Arn JL. Fixed dilated pupil (Urrets-Zavalia syndrome) and anterior subcapsular cataract formation after Descemet stripping endothelial keratoplasty. Cornea 2009; 28(10):1184-1186. 10. Russell HC, Srinivasan S. Urrets-Zavalia syndrome following Descemets stripping endothelial keratoplasty triple procedure. Clin Exp Ophthalmol. 2011; 39(1):85-87. 11. Park SH, Kim SY, Kim HI, Yang SW. Urrets-Zavalia syndrome following iris-claw phakic intraocular lens implantation. J Refract Surg. 2008; 24(9):959-961. 12. Aralikatti AK, Tomlins PJ, Shah S. Urrets-Zavalia syndrome following intracameral C3F8 injection for acute corneal hydrops. Clin Exp Ophthalmol. 2008; 36(2):198-199. 13. Yuzbasioglu E, Helvacioglu F, Sencan S. Fixed, dilated pupil after phakic intraocular lens implantation. J Cataract Refract Surg. 2006; 32(1):174-176. 14. Spadea L, Viola M, Viola G. Regression of Urrets-Zavalia syndrome after deep lamellar keratoplasty for keratoconus: a case study. Open Ophthalmol J. 2008; 2:130-131.

B. Facial synkinesis: abnormal facial movements resulting from aberrant regeneration of the facial nerve after an insult such as Bell palsy, trauma, or surgery (ie, resection of a vestibular schwannoma). C. Hemifacial spasm: involuntary, very rapid, brief contraction of facial muscles resulting from abnormal stimulation of the facial nerve. This is usually either idiopathic or due to compression of the facial nerve by a vascular loop. Movements may be subtle and limited to a small region or very severe and involve most facial muscles. Neuroimaging of the brainstem with attention to the origin and course of the facial nerve is required to identify the rare cases that have a mass lesion causing hemifacial spasm. D. Epilepsy partialis continua: involuntary, very rapid, brief contraction of facial muscles resulting from focal seizure activity. These facial movements may be indistinguishable from hemifacial spasm, but other features such as hand involvement or mental status changes are helpful in making the diagnosis. E. Facial myokymia: often described as having the appearance of a bag of worms, these involuntary facial movements are undulating or rippling movements just beneath the skin and involve facial regions outside of the eyelid. III. Etiology and Localization of Facial Myokymia A. Facial myokymia is most often due to a lesion in the pons at the level of the facial nerve nucleus and/ or fascicles. While an unusual finding, it most often represents a serious neurological condition such as a pontine glioma or a demyelinating lesion. Other reported causes include neurocysticercosis, radiation, multiple systems atrophy, and Guillain-Barre syndrome. B. When due to a brainstem lesion, facial myokymia may occur in isolation or may be associated with other neuro-ophthalmologic examination findings such as facial weakness, ocular motor dysfunction, or nystagmus. IV. Summary and Conclusions A. Most causes of a twitching eyelid are benign and self-limited. B. With careful clinical evaluation, the following redflags can be identified: 1. Eyelid myokymia that also involves other facial muscles 2. Eyelid or facial myokymia associated with other neuro-ophthalmologic abnormalities C. Further workup is indicated for red-flag cases. This usually involves neuroimaging and referral to a neuro-ophthalmologist or neurologist.

Case #13: The Twitching Eyelid Whats the Cause?


Nathan T Tagg MD
Answer and Teaching Points
I. Introduction A. Unilateral eyelid twitching is a common phenomenon among adults of all ages. The lower eyelid is more commonly involved than the upper eyelid. B. The vast majority of cases are benign and selflimited. C. Occasionally, eyelid twitching can indicate a serious neurological disease. D. The purpose of this presentation is to highlight features of eyelid and facial movements that require further investigation. II. Differential Diagnosis of Abnormal Unilateral Facial Movements A. Benign eyelid twitching: small, brief fasciculationlike movements of the orbicularis oculi muscle. These are most often self-limited and infrequent and may be triggered by stress, caffeine intake, and other potentially modifiable entities. Occasionally, movements may be undulating or rippling and when limited to the eyelid, this is referred to as eyelid myokymia. Chronic eyelid myokymia is also most likely to be benign.

2011 Subspecialty Day

Neuro-Ophthalmology

Section II: What Is Wrong With This Eyelid/Pupil?

29

References
1. Bhatia R, Desai S, Garg A, et al. Isolated facial myokymia as a presenting feature of pontine neurocysticercosis. Mov Disord. 2008; 23(1):135-137. Case, images, and video used with permission. 2. Banik R, Miller NR. Chronic myokymia limited to the eyelid is a benign condition. J Neuroophthalmol. 2004; 24(4):290-292. 3. Rubin M, Root JD. Electrophysiologic investigation of benign eyelid twitching. Electromyogr Clin Neurophysiol. 1991; 31(6):377378. 4. Rad EW, Skorpil V, Kaeser HE. Facial myokymia. Eur Neurol. 1975; 13(6):499-512. 5. Sharma RR, Mathad NV, Joshi DN, Mazarelo TB, Vaidya MM. Persistent facial myokymia: a rare pathognomonic physical sign of intrinsic brain-stem lesions: report of 2 cases and review of literature. J Postgrad Med. 1992; 38(1):37-40, 40A-40B. 6. Keane JR. Cysticercosis: unusual neuro-ophthalmologic signs. J Clin Neuroophthalmol. 1993; 13(3):194-199.

30

Section III: What Is Wrong With This Eye Movement?

2011 Subspecialty Day

Neuro-Ophthalmology

Section III: What Is Wrong With This Eye Movement? Answer and Teaching Points
Case #14: Fourth Nerve Palsy
Mark L Moster MD
Answer and Discussion
The classic teaching is that an isolated fourth nerve palsy is benign and very often due to worsening of a congenital fourth nerve palsy. Often reviewing old photos will show a persistent head tilt since childhood. Another clue is that there is an increased vertical fusional capacity of greater than 6 prism diopters. What is wrong with this picture in our case for a congenital fourth nerve palsy? The vertical fusional capacity was not increased, and review of photos did not reveal a prior head tilt. Review of the MRI demonstrated a lesion on the fourth nerve in its subarachnoid course, consistent with a trochlear nerve schwannoma. In this case the differential diagnosis of all underlying causes of fourth nerve palsy as well as mimickers of fourth nerve palsy must be considered. I. Causes of Fourth Nerve Palsy A. Congenital 1. Head tilt noted as a child 2. Increased vertical fusional capacity 3. Torsional component is not symptomatic. B. Trauma, often bilateral 1. Alternating hypertropia in different positions of eyes and head 2. Excyclotorsion > 10 degrees 3. V pattern vertical deviation C. Vasculopathic: Risk factors 1. Diabetes 2. Hypertension 3. Hyperlipidemia 4. Smoking D. Demyelinating disease: Less frequent than sixth or third nerve palsy, because of short intramedullary course in brainstem E. Cerebral infarct, hemorrhage, or other structural lesion F. Peripheral compressive lesion 1. Perimesencephalic cistern/ subarachnoid space 2. Cavernous sinus 3. Orbit G. Herpes zoster H. Meningitis II. Mimickers of Fourth Nerve Palsy A frequent clue is that the ocular deviation does not fit perfectly in the 3-step test or that the deviation is worse in upgaze than downgaze. Excyclotorsion may not be present, either. A. Myasthenia gravis 1. Fatigability 2. Variability 3. Associated ptosis B. Thyroid ophthalmopathy 1. Associated orbital signs 2. Hypertropia is usually worse in upgaze since it is due to inferior rectus restriction. 3. IOP elevates more than 3 mm Hg in upgaze. C. Skew deviation 1. Caused by a lesion in posterior fossa 2. Vertical deviation, which may be comitant or incomitant 3. Ocular tilt reaction is the type of skew that most often mimics a fourth nerve palsy. a. Hypotropia b. Head tilt toward hypotropic eye c. Instead of excyclotorsion of hypertropic eye there is bilateral torsion toward the hypotropic eye.

References
1. Mollan SP, Edwards JH, Price A, Abbott J, Burdon MA. Aetiology and outcomes of adult superior oblique palsies: a modern series. Eye 2009; 23(3):640-644. 2. Brazis PW. Isolated palsies of cranial nerves III, IV, and VI. Semin Neurol. 2009; 29(1):14-28. 3. Bennett JL, Pelak VS. Palsies of the third, fourth, and sixth cranial nerves. Ophthalmol Clin North Am. 2001; 14(1):169-185, ix. 4. Elmalem VI, Younge BR, Biousse V, Leavitt JA, Moster ML, et al. Clinical course and prognosis of trochlear nerve schwannomas. Ophthalmology 2009; 116(10):2011-2016. 5. Moster ML, Bosley TM, Slavin ML, Rubin SE. Thyroid ophthalmopathy presenting as superior oblique paresis. J Clin Neuroophthalmol. 1992; 12(2):94-97. 6. Wong AM. Understanding skew deviation and a new clinical test to differentiate it from trochlear nerve palsy. J AAPOS. 2010; 14(1):61-67.

2011 Subspecialty Day

Neuro-Ophthalmology

Section III: What Is Wrong With This Eye Movement?

31

Case #15: Ocular Flutter


Rod Foroozan MD
Answer and Teaching Points
Questions for panel What is this eye movement disorder, and how is it different than opsoclonus? Is this nystagmus, and if not, why not? Could this be voluntary nystagmus? What are the most common causes of ocular flutter? Other than treating the underlying condition, is there any treatment that will help the abnormal ocular oscillations? Summary Ocular flutter is a rare disorder characterized by back-to-back saccades. The abnormal ocular oscillations are believed to result from dysfunction of the pause/ burst cells, which govern saccade generation, in the pons. The most common causes can broadly be grouped into inflammatory, including sterile (such as demyelination), and infectious (such as post viral) and paraneoplastic categories. Most patients with an infectious cause show spontaneous improvement weeks to months after the onset of symptoms.

Patient education/ awareness of signs of systemic myasthenia Development of bulbar symptoms, dyspnea, dysarthria Medications that may affect neuromuscular transmission Myasthenic crisis must be addressed emergently. Preoperative consideration of possible anesthetic effect on ocular symptoms Clinical Signs and Symptoms Differentiating Myasthenia From Strabismus Intermittent binocular diplopia: Variable ocular misalignment Brow elevation, chin up head posture due to ptosis Fluctuation in severity May be asymmetric Diurnal variation in ocular symptoms, with worsening in the evening Weakness of orbicularis oculi on exam Fatigability: Elicited on prolonged upgaze, symptoms worsen with fatigue Look for eyelid fasciculations with saccades Cogan lid twitch, while suggestive of myasthenia, is neither diagnostic nor specific. Diagnosis of Myasthenia Clinical diagnosis may be supported by: Acetylcholine receptor antibody (ARAs) serologies present in 50+% of purely ocular myasthenia present in ~90% of generalized myasthenia Binding, modulating, blocking antibodies MuSK Antibodies: Muscle-specific kinase may be abnormal in myasthenia. Associated with bulbar symptoms Rarely found in purely ocular myasthenia EMG testing: Repetitive nerve stimulation results in decremental action potential. Single fiber EMG: Jitter occurs in disorders of neuromuscular transmission. Edrophonium testing: Proper precautions should be in place in case of bradycardia, particularly in cardiac patients. Neostigmine testing: Intramuscular administration is easier in children; no need for IV. Ice pack and rest testing for ptotic patients Noninvasive test, easy to administer, with low morbidity Retrospective studies show reproducibility, sensitivity (80%), and specificity. However, the precise mechanism of action is unknown. Neuroimaging: Consider MRI brain and orbits (dedicated fat suppressed post gad) if: Progressive, painless ophthalmoplegia Cranial neuropathy or central process not excluded AAA = Anisocoria , afferent pupillary defect, aneurysm (suspicion necessitates MRA/CTA head) Obtain chest imaging (CT/MR) to evaluate for thymoma in all cases. Medical Management Ocular symptom relief Pyridostigmine: acetylcholinesterase inhibitor Eyelid crutches or patching

References
1. Rosenfeld MR, Dalmau J. Update on paraneoplastic and autoimmune disorders of the central nervous system. Semin Neurol. 2010; 30:320-331. 2. Waisbourd M, Kesler A. Postinfectious ocular flutter. Neurology 2009; 72:1027. 3. Yee RD, Spiegel PH, Yamada T, Abel LA, Suzuki DA, Zee DS. Voluntary saccadic oscillations, resembling ocular flutter and opsoclonus. J Neuroophthalmol. 1994; 14:95-101.

Case #16: Exophoria, but Wait Thats Fatigable?


Gabrielle Bonhomme MD
Answer and Teaching Points
Myasthenia Gravis Humorally mediated autoimmune disorder of neuromuscular transmission associated with a decreased number of intact skeletal muscle acetylcholine receptors Prevalence of approximately 50-125 cases per million Bimodal distribution (young women and men over 60) May occur in children, and more rarely, neonates 90% of myasthenic patients will exhibit ocular symptoms/ involvement at some point. Ocular Myasthenia 50% of all myasthenic patients initially present with purely ocular symptoms, usually ptosis or diplopia. Roughly 50% of those patients may develop generalized (systemic) disease, the great majority within 2-3 years of symptom onset.

32

Section III: What Is Wrong With This Eye Movement? Immunosuppression/ immunomodulation Corticosteroids: Good efficacy reported in ocular myasthenia, but side effects of chronic use may complicate course. Mycophenolate mofetil: adjuvant, steroid-sparing Azathioprine: Slow effect; monitor BM, liver function tests Cyclosporine: Slow effect; monitor BP, renal function Refractory or generalized myasthenia may require hospitalization for: IV gammaglobulin Plasmapheresis Thymectomy: Potential for cure * 60% of surgical candidates with generalized symptoms may experience remission. * Thymoma is considered an indication for surgery. * Surgical morbidity is decreased with less invasive, transcervical procedures.

2011 Subspecialty Day

Neuro-Ophthalmology

11. Mulder DG, Herrmann C, Keesey JC, et al. Thymectomy for myasthenia gravis. Am J Surg. 1983; 146:61-66. 12. Walsh FB. Myasthenia gravis and its ocular signs: a review. Trans Am Ophthalmol Soc. 1943; 41:556-624.

Case #17: Spasmus Nutans... How Do I Know?


Peter A Quiros MD
Answer and Teaching Points
Spasmus nutans is a well-defined triad of: Head bobbing Head tilt (so-called torticollis) Dissociated nystagmus It typically appears in the first or second year of life and resolves spontaneously. It is not associated with any typical or underlying pathology when isolated and seen in its entirety.1 Pseudospasmus nutans, formerly atypical spasmus nutans, is often used to describe a dissociative nystagmus that presents with or without the head tilt and/or bobbing (ie, the classic triad is absent) or when the onset occurs in older children.2 Pseudospasmus nutans, unlike true spasmus nutans, is often associated with anatomic lesionsin particular those involving the chiasm, hypothalamus, and brainstem. There are many such examples in the literature; most describe gliomas that involve the chiasm, hypothalamus, or brainstem.3-5 Pseudospasmus nutans has also been reported in infectious and demyelinating syndromes (lyme and multiple sclerosis, respectively), as well as paraneoplastic syndromes.6 The key to making the diagnosis is to have a high index of suspicion in all cases that: Present outside the characteristic age range (1-2 years of age) and/or Do not exhibit all 3 features of the triad and/or Do not present as an isolated entity (accompanied by vision loss, ataxia, or other focal neurologic findings) All such patients should be imaged, as the vast majority of causes will be visible on neuroimaging. MRI is the preferred modality in order to better assess the chiasm and brainstem. If neuroimaging does not reveal an anatomic lesion, evidence of demyelination, or other abnormality, then a laboratory workup may be considered as appropriate. However, the latter are very rare and, again, the majority of etiologies in cases of pseudospasmus nutans will be revealed with imaging of the brain. The question Why not image all patients? does arise, but one must keep in mind that the majority of patients are children and that scanning children is not as simple as scanning adults. CT exposes them to unnecessary radiation at young ages and most require sedation for MRI. It is estimated that if all children with spasmus nutans were to be scanned there would be somewhere between 5000 and 10,000 unnecessary scans conducted on children per year.7 As ophthalmologists we can avoid unnecessary diagnostic workup by carefully examining all patients for vision loss and following-up on initially normal examinations to ensure that vision loss has not developed. Referral to a neurologist or

Take Home Points Ocular myasthenia often mimics the pattern of a cranial neuropathy or decompensated phoria. Look for a history of intermittent symptoms, diurnal variation, fatigability, and fluctuations in severity. Pupillary abnormalities should not occur in isolated ocular myasthenia, and they warrant neuroimaging and further evaluation to exclude compressive lesions. Monitor and educate patients for signs of generalization.

References
1. Witoonpanich R, Dejthevapom C, Sriphrapradang A, Pulkes T. Electrophysiological and immunological study in myasthenia gravis: diagnostic sensitivity and correlation. Clin Neurophysiol. Epub ahead of print 16 Mar 2011. PMID: 21419697. 2. Pineles SL, Avery RA, Moss HE, et al. Visual and systemic outcomes in pediatric ocular myasthenia gravis. Am J Ophthalmol. 2010; 150(4):453-459. 3. Antonio-Santos AA, Eggenberger ER. Medical treatment options for ocular myasthenia gravis. Curr Opin Ophthalmol. 2008; 19(6):468-478. 4. Benatar M, Kaminski HJ. Evidence report: the medical treatment of ocular myasthenia (an evidence based review). Neurology 2007; 68(24):2144-2149. 5. Van Stavern GP, Bhatt A, Haviland J, Black EH. A prospective study assessing the utility of Cogans lid twitch sign in patients with isolated unilateral or bilateral ptosis. J Neurol Sci. 2007; 256:84-85. 6. SFEMG in ocular myasthenia gravis diagnosis. Clin Neurophysiol. 2000; 111(7):1203-1207. 7. Detection and characterization of MuSK antibodies in seronegative myasthenia gravis. Ann Neurol. 2004; 55(4):580-584. 8. Monsul NT, Patwa HS, Knorr AM, et al. The effect of prednisone on the progression from ocular to generalized myasthenia gravis. JNeurol Sci. 2004; 217:131-133. 9. Kubis KC, Danesh-Meyer HV, Savino PJ, et al. The ice test versus the rest test in myasthenia gravis. Ophthalmology 2000; 107:19951998. 10. Golnik KC, Pena R, Lee AG, et al. An ice test for the diagnosis of myasthenia gravis. Ophthalmology 1999; 106:1282-1286.

2011 Subspecialty Day

Neuro-Ophthalmology

Section III: What Is Wrong With This Eye Movement?

33

neuroophthalmologist for complete neurologic testing should also be conducted.

References
1. Norton EWD, Cogan DG. Spasmus nutans: a clinical study of 20 cases followed 2 years or more since onset. Arch Ophthalmol. 1954; 52:442-446. 2. Brodsky MC. Nystagmus in Children in Pediatric Neuro-ophthalmology. Springer, 2010, 383-441. 3. Newman SA, Hedges TR, Wall M, et al. Spasmus nutans: or is it?. Surv Ophthalmol. 1990; 34: 453-456. 4. Kelly TW. Optic glioma presenting as spasmus nutans. Pediatrics 1970; 45:295-296. 5. Albright AL, Sclabassi RJ, Slamovits TL, et al. Spasmus nutans associated with optic gliomas in infants. J Pediatrics. 1984; 105:778-780. 6. Kiblinger GD, Wallace BS, Hines M, et al. Spasmus nutans-like nystagmus is often associated with underlying ocular, intracranial, or systemic abnormalities. J Neuroophthalmol. 2007; 27:118-122. 7. King RA, Nelson LB, Wagner RS. Spasmus nutans: a benign clinical entity. Arch Ophthalmol. 1986; 104:1501-1504.

myasthenia. Continued observation often will reveal the diagnosis: within days or weeks the myasthenic patient may reveal diurnal fatigue, new ptosis, or a major change in the pattern of eye movements.

References
1. Argyriou AA, Karanasios P, Potsios C, et al. Myasthenia gravis initially presenting with pseudo-internuclear ophthalmoplegia. Neurol Sci. 2009 Oct;30(5):387-8. 2. Khanna S, Liao K, Kaminski HJ, et al. Ocular myasthenia revisited: insights from pseudo-internuclear ophthalmoplegia. J Neurol. 2007 Nov;254(11):1569-74. 3. Ito K, Mizutani J, Murofushi T, et al. Bilateral pseudo-internuclear ophthalmoplegia in myasthenia gravis. ORL J Otorhinolaryngol Relat Spec. 1997 Mar-Apr;59(2):122-6. 4. Jay WM, Nazarian SM, Underwood DW. Pseudo-internuclear ophthalmoplegia with downshoot in myasthenia gravis. J Clin Neuroophthalmol. 1987 Jun;7(2):74-6.

Case #19: Microvascular Third Nerve Palsy, but Whats Up With That Lid?
Beau Benjamin Bruce MD MS
Answer and Teaching Points
The new findings, incomplete lowering of the eyelid on infraduction and elevation of the eyelid on adduction, are signs of aberrant regeneration (also called synkinesis) of the third cranial nerve. These findings indicate that this is not a microvascular third nerve palsy. Instead, these findings are indicative of a partial injury to the nerve from compression (eg, a tumor or aneurysm) or trauma. The patient has no history of trauma. Thus, it is likely that a mass lesion has been overlooked during the initial evaluation. In this case, careful review of the original neuroimaging study revealed a subtle asymmetry of the cavernous sinus on the right. A repeat study with contrast demonstrated a homogenously enhancing lesion with a dural tail in the lateral right cavernous sinus suggestive of a meningioma. Aberrant regeneration is a phenomenon that can occur in most, if not all, peripheral nerves. Another example, not infrequently seen in ophthalmology, is hyperlacrimation (crocodile tears) seen after facial (seventh cranial) nerve palsy. The third cranial nerve also has particularly interesting manifestations when aberrant regeneration occurs because of the large number of structures that it innervates. While this patient showed two signs of aberrant regeneration of the third cranial nerve, elevation of the upper eyelid with attempted downgaze (pseudo-Graefe sign) and elevation of the eyelid on attempted adduction, several others are possible: segmental constriction of the pupil with movement in direction of eye muscles normally innervated by the third cranial nerve, retraction of the globe with attempted vertical gaze (presumably due to co-contraction of the superior and inferior recti), and adduction of the eye with attempted vertical gaze. Aberrant regeneration typically takes 2 or 3 months to develop following a partial injury to the nerve, regardless of

Case #18: Slowed Saccade and a ? Internuclear Ophthalmoplegia


Julie Falardeau MD
Answer and Teaching Points
An ice test was performed with complete resolution of his ptosis. Acetylcholine receptor antibody testing was positive, confirming the diagnosis of myasthenia gravis. Internuclear ophthalmoplegia (INO) is typically caused by a demyelinating lesion in young patients and by an ischemic lesion in older patients. The major features of INO include (1) limited or slowed adduction and (2) abduction overshoot and nystagmus in the contralateral eye. The minor signs of INO include (1) skew deviation, (2) vertical nystagmus, (3) impaired convergence, (4) impaired vertical pursuit, and (5) abnormal vertical vestibuloocular reflex. The principal difficulty in diagnosing INO lies in failure to appreciate mild, or even moderate, slowing of adduction saccades in the presence of a full range of motion. The minor signs are inconsistently present and depend on the completeness of injury to the medial longitudinal fasciculus and whether one or both sides are involved. The absence of a lesion in the medial longitudinal fasciculus on MRI should raise a suspicion for pseudo-INO. Infranuclear pseudo-INO can mimic perfectly the major signs and may even simulate the minor signs of INO, as seen occasionally in myasthenia. Most myasthenic patients presenting with pseudo-INO will have associated myasthenic signs/symptoms such as ptosis or proximal muscle weakness. For the rare patient with pseudoINO as the sole apparent presenting sign of myasthenia, there are a number of neuro-ophthalmologic maneuvers to elicit masked weakness, including evoking lid or saccadic fatigue, eliciting the lid twitch sign, and demonstrating the rapid eye movements of

34

Section III: What Is Wrong With This Eye Movement?

2011 Subspecialty Day

Neuro-Ophthalmology

cause, and would not be expected at the time of presentation. However, primary aberrant regeneration, ie, without an obvious preceding paretic phase, is occasionally observed from the effect of very slow-growing mass lesions. Because ischemic causes (ie, microvascular, frequently associated with diabetes and hypertension) rarely, if ever, cause aberrant regeneration, a careful search for alternative causes must be undertaken whenever it is observed. The pathophysiology of aberrant regeneration is generally assumed to result from misdirection of regenerating fibers after partial injury to the peripheral portion of the nerve. However, theories involving central mechanisms have also been proposed. The treatment of a patients residual ptosis and diplopia are complicated substantially by the presence of aberrant regeneration because the abnormal co-action of the muscles in synkinesis typically leads to difficult-to-correct lid asymmetry and narrowly limited regions of single vision. Various surgical techniques can be considered, but there must be a careful discussion with the patient to manage their expectations before proceeding with surgical therapy. In conclusion, aberrant regeneration of the third ocular nerve is a key red flag in neuro-ophthalmology that indicates, in the absence of a history of significant trauma, that the palsy is due to a more serious underlying lesion such as an aneurysm or tumor.

1. Often worse in morning 2. Normal saccades E. Proptosis 1. Can be axial or nonaxial 2. Associated with increased resistance to retropulsion F. Retrobulbar ache 1. Worse with eye movement 2. Degree of pain can confuse clinical diagnosis. G. IOP elevation in primary often worsened with upgaze or abduction H. Pulsatile tinnitus possible during hyperthyroid phase II. Allergies/Sinusitis A. Conjunctival injection and chemosis 1. Caruncle particularly hyperemic and edematous 2. Caruncle often the epicenter of itch. B. Periorbital edema and erythema 1. Often more prominent in morning 2. Lower eyelid edema especially common with maxillary sinusitis C. No lid retraction, lid lag, or lagophthalmos D. Diplopia is extremely rare. E. Enophalmos associated with silent sinus syndrome. No proptosis. F. Retrobulbar ache accompanies ache over involved sinus. G. No alterations in IOP H. No tinnitus III. Orbital Inflammatory Syndrome Nonspecific vs. specific entities associated with systemic disease (eg, sarcoidosis, Wegener granulomatosis, IgG disease). A. Conjunctival injection and chemosis: Caruncle not particularly hyperemic and edematous B. Periorbital edema and erythema: Often more prominent in morning C. Lid retraction not typical even in sclerosing variant. D. Diplopia may be due to active myositis or sclerosing restriction. 1. Often worse in morning in active myositis 2. Constant with potential for progression in sclerosing variant 3. Normal saccades E. Proptosis more common, but enophalmos possible with sclerosing subvariant. 1. Can be axial or nonaxial

References
1. Bruce BB, Biousse V, Newman NJ. Third nerve palsies. Semin Neurol. 2007; 27:257-268. 2. Goonetilleke A, Yuill GM. Neurological picture: aberrant regeneration of the third cranial nerve. J Neurol Neurosurg Psychiatry. 1996; 60:281. 3. Miller NR, Newman NJ, Biousse V, Kerrison JB, eds. Walsh and Hoyts Clinical Neuro-Ophthalmology. 6th ed. Philadelphia: Lippincott Williams & Wilkins; 2005. 4. Lepore FE, Glaser JS. Misdirection revisited: a critical appraisal of acquired oculomotor nerve synkinesis. Arch Ophthalmol. 1980; 98:2206-2209. 5. Weber ED, Newman SA. Aberrant regeneration of the oculomotor nerve: implications for neurosurgeons. Neurosurg Focus. 2007; 23:E14.

Case #20: Thyroid Eye Disease... or Not?


Kimberly Cockerham MD FACS
Answer and Teaching Points
I. Thyroid Eye Disease Manifestations A. Conjunctival injection and chemosis: Caruncle particularly hyperemic and edematous B. Periorbital edema and erythema, often more prominent in morning C. Lid retraction D. Diplopia

2011 Subspecialty Day

Neuro-Ophthalmology

Section III: What Is Wrong With This Eye Movement? F. Retrobulbar ache 1. Worse with eye movement 2. Degree of pain can confuse clinical diagnosis. G. IOP elevation in primary often worsened with upgaze or abduction H. Pulsatile tinnitus VI. TED Plus Myasthenia Gravis

35

2. Associated with increased resistance to retropulsion F. Severe pain characteristic of active NSOI; retrobulbar ache may be present in sclerosing variant. 1. Scleritis, myositis, and optic neuritis worse with eye movement 2. Degree of pain can confuse clinical diagnosis. G. IOP elevation in primary often worsened with upgaze or abduction H. No tinnitus IV. Carotid-Cavernous Fistula A. Conjunctival injection and chemosis 1. Caruncle not particularly hyperemic and edematous 2. Cork-screw vessels touch the limbus. B. Periorbital edema and erythema: Present throughout the day C. No lid retraction; ptosis possible D. Diplopia 1. Constant cranial nerve III, IV, and/ or VI 2. Slowed saccades 3. Not worse in the morning E. Proptosis 1. Typically axial 2. Associated with increased resistance to retropulsion F. Retrobulbar ache 1. Worse with eye movement 2. Degree of pain can confuse clinical diagnosis. G. IOP elevation in primary, often worsened with upgaze or abduction H. Pulsatile tinnitus characteristic V. Thyroid Eye Disease Plus: The Ultimate Challenge Any or all of the signs of thyroid eye disease (TED): A. Conjunctival injection and chemosis: Caruncle particularly hyperemic and edematous B. Periorbital edema and erythema: Often more prominent in morning C. Lid retraction D. Diplopia 1. Often worse in morning 2. Normal saccades E. Proptosis 1. Can be axial or nonaxial 2. Associated with increased resistance to retropulsion

A. Conjunctival injection and chemosis may be absent. B. Periorbital edema and erythema may be absent. C. Eyelid retraction possible as response to the characteristic ptosis. 1. Difficulties with sustained upgaze 2. Cogan lid twitch D. Diplopia 1. Variable 2. Worse with use; fatigue common 3. Intrasaccadic or hypometric saccades E. No proptosis F. No ache or pain G. No alteration in IOP in primary or with gaze H. Systemic features (proximal muscle weakness, difficulty swallowing or breathing) VII. TED Plus Lymphoma A. Conjunctival injection and chemosis may be absent. B. Periorbital edema and erythema may be absent. C. Eyelid retraction possible as response to the characteristic ptosis D. Lacrimal gland enlargement common E. Diplopia 1. Constant depends on muscle infiltrated; tendon involved on imaging 2. Normal saccadic velocity despite limited pursuits F. Proptosis and increased resistance to retropulsion typical G. No ache or pain H. No alteration in IOP in primary or with gaze VIII. TED Plus Cranial Nerve Palsy A. Conjunctival injection and chemosis may be present or absent. B. Periorbital edema and erythema may be present or absent. C. Eyelid retraction possible as response to the characteristic ptosis of CN III D. Diplopia 1. Acute onset

36

Section III: What Is Wrong With This Eye Movement? 2. Constant 3. Slowed saccades E. Proptosis and resistance to retropulsion may be present or absent. F. Pain may be present in some CN III palsy patients. G. No alteration in IOP in primary or with gaze H. Systemic risk factors: hypertension, diabetes, hypercholesterolemia

2011 Subspecialty Day

Neuro-Ophthalmology

Selected Readings
1. Cockerham KP, Chan SS. Thyroid eye disease. Neurologic Clin. 2010; 28:729-755. 2. Char DH. Thyroid eye disease. Br J Ophthalmol. 1996; 80:922926. 3. Bartalena L, Pinchera A, Marcocci C. Management of Graves ophthalmopathy: reality and perspectives. Endocr Rev. 2000; 168-199. 4. Denniston A, Dodson P, Reuser T. Diagnosis and management of thyroid eye disease. Hosp Med. 2002; 63(3):153-156. 5. Kennerdell JS, Cockerham KP, Maroon JP, Rothfus W. Dysthyroid orbitopathy. In: Kennerdell JS, Cockerham KP, eds. Practical Diagnosis and Management of Orbital Disease. Boston: Butterworth/ Heinemann Press, 2001: 53-75.

2011 Subspecialty Day

Neuro-Ophthalmology

Section IV: What Is Wrong With This Test?

37

Section IV: What Is Wrong With This Test? Answer and Teaching Points
Case #21: Potential Sources of Error in ERG or mfERG Testing
Randy Kardon MD PhD
Answer and Teaching Points
This patient was eventually diagnosed as having cancer-associated autoimmune retinopathy (CAR), after it was determined that the Ganzfeld ERG was artifactually interpreted as being normal but was really flat, due to a photomyoclonic reflex potential superimposed on the recording. The light-induced electromyogram (EMG) response was studied extensively by Johnson and Massof in 1982 in relation to the ERG and the potential artifact that it may cause.1 The photomyoclonic potential is elicited from the orbicularis oculi muscles in response to light stimuli having the intensity and duration typically used for ERG recordings.2-5 It was found that the light-induced photomyoclonic potential had a mean latency of 59 msec 7 msec. Similar to the ERG, the photomyoclonic evoked potential has an initial negative potential that peaks at about 100 msec and then a positive potential that peaks around 300-350 msec, which may confound the interpretation of the Aand B-wave components of the ERG, as occurred in the patient presented here with CAR. The photomyoclonic potential falls within the timeframe where an ERG potential is expected to be measured. This is why it may be confused with the evoked potentials coming from the retina as the ERG. The photomyoclonic voltage from the eyelid muscles evoked in response to a flash of light does not require the eyelid to actually blink. It is commonly seen in ERG recordings where the eyelids are held open with a Burian-Allen corneal ERG electrode. The photomyoclonic potential may also be recorded at intensities of light below the level required to elicit a reflex blink. Since the light-induced electromyogram has a direct and consensual component, it is present in either eye upon receiving light input from one or both eyes. Patients with CAR often complain of photosensitivity, making them even more likely to produce a sizable photomyoclonic potential that may add to or be confused with an ERG potential. With repeated flashes of light, there is some evidence for habituation of this light-induced EMG reflex potential.1 In addition to the photomyoclonic potential coming from the orbicularis muscle, Johnson and Massof also provided evidence for a slight downward and adducting movement of the eye in response to the ERG light stimulus, and the resulting electro-oculogram (EOG) potentials may also interfere with the correct interpretation of the ERG. When the pupils are not dilated, the movement of the iris in response to light also can contribute to a potential that occurs at the same time as the late component of the ERG. Recently, evidence was published implicating input from the intrinsically sensitive melanopsin retinal ganglion cells as a major mediator of photodynia (discomfort upon light exposure) in patients with light sensitivity and photoreceptor degenerations.6 The emerging evidence supports that contention that the melanopsin-containing retinal ganglion cells may mediate lightinduced responses (pupil contraction, eyelid EMG, and discomfort) even in patients with severe photoreceptor degenerations. Our own research has shown that patients blind or nearly blind from severe rod and cone loss may still show large pupil contractions to bright blue light, which stimulates intrinsic melanopsin activation.7-9 It is possible that the photomyoclonic potentials from the eyelid muscles may also be mediated by afferent input from these special retinal ganglion cells10 and could explain why such potentials could be mistaken for a recordable ERG, as occurred in the patient example presented here. In CAR, retinal degeneration usually occurs in the outer retina in the presence systematic cancer (most commonly adenocarcinoma, but other types of cancer may also incite CAR). The most common cancers found with CAR include small cell lung carcinoma, breast cancer, and gynecologic cancer. CAR has also been found in other types of lung cancer, colon cancer, melanoma, skin squamous cancer, kidney cancer, pancreatic, lymphoma, basal cell tumor, and prostate cancer. Antibodies against retinal tissue can be detectable in serum in many, but not all cases. In many instances, a patients visual loss from CAR precedes the diagnosis of cancer. Constant photopsias are thought to be due to spontaneous electrical discharge from photoreceptors that are being affected by the autoimmune process. The presence of constant photopsias in the presence of visual field loss in both eyes is highly suspicious of CAR, and the ERG is usually either severely depressed or nonrecordable under photopic or scotopic conditions (or both). In patients in whom serum antibodies to retinal tissue have been detected, a number of different antibodies have been isolated against many specific retinal proteins, but usually there is one predominant type of antibody found in a given patient, which may be an antibody to any of the following retinal proteins: recoverin, carbonic anhydrase, trasducin, alpha-enolase, TULP1, PNR photoreceptor cell-specific nuclear receptor, heat shock protein HSC 70, and arrestin. There are many more antibodies against different retinal proteins that are not yet identified. Anti-recoverin antibodies, which are the most common antibodies associated with CAR, usually present with acute severe vision loss and paracentral or equatorial scotoma. CAR with anti-enolase antibodies causes cone dysfunction, typically leading to asymmetric central vision loss with slower progression. It is beyond the scope of this discussion to thoroughly review the clinical presentation, diagnosis, and treatment of CAR. However, the main learning point of the case presented was that each piece of information needs to be scrutinized carefully when developing a diagnosis compatible with the patients symptoms and findings. In this case, the ERG was initially assumed to be normal based on an interpretation that was confounded by the electrical potentials recorded during the ERG, which turned out to be mainly from the eyelid orbicularis muscle evoked in response to light input and not from the rods and cones.

38

Section IV: What Is Wrong With This Test?

2011 Subspecialty Day

Neuro-Ophthalmology

References
1. Johnson MA, Massof RW. The photomyoclonic reflex: an artefact in the clinical electroretinogram. Br J Ophthalmol. 1982; 66(6):368-378. 2. Mukuno K, Aoki S, Ishikawa S, et al. Three types of blink reflex evoked by supraorbital nerve, light flash and corneal stimulations. Jpn J Ophthalmol. 1983; 27(1):261-270. 3. Malin JP. The optically evoked orbicularis oculi reflex: method, normal values, habituation [in German]. EEG EMG Z Elektroenzephalogr Elektromyogr Verwandte Geb. 1982; 13(3):101-107. 4. Yasuhara M, Naito H. Potential changes and eyelid microvibration elicited by flash stimulation. Int J Neurosci. 1982; 17(1):23-31. 5. Ozaki T. The physiologic properties of potential changes elicited in the eyelid by flash stimulation [authors translation to Japanese]. Nippon Seirigaku Zasshi. 1976; 38(1):15-16. 6. Noseda R, Kainz V, Jakubowski M, et al. A neural mechanism for exacerbation of headache by light. Nat Neurosci. 2010; 13(2):239245. 7. Kawasaki A, Kardon RH. Intrinsically photosensitive retinal ganglion cells. J Neuroophthalmol. 2007; 27(3):195-204. 8. Kardon R, Anderson SC, Damarjian TG, Grace EM, Stone E, Kawasaki A. Chromatic pupil responses: preferential activation of the melanopsin-mediated versus outer photoreceptor-mediated pupil light reflex. Ophthalmology 2009; 116(8):1564-1573. 9. Kardon R, Anderson SC, Damarjian TG, Grace EM, Stone E, Kawasaki A. Chromatic pupillometry in patients with retinitis pigmentosa. Ophthalmology 2011; 118(2): 376-381. 10. Kardon R, Anderson S, Full J, Poolman P. The light induced electromyogram (EMG): a reflex pathway integrating the melanopsin retinal ganglion cell, trigeminal sensory nucleus and facial nerve. Association for Research in Vision and Ophthalmology, IOVS abstract, May 2011.

Teaching Points
Re-read the MRI to be sure the correct scan was completed and that the report was correct. Gadolinium can be helpful. If the MRI is unremarkable, seek other causes for bitemporal hemianopia (optic nerve, retina, nonorganic, refractive, reliable testing, artifacts). Seek other testing to confirm diagnosis (OCT, mfERG). If bitemporal hemianopia respects the vertical meridian, the lesion is chasmal in origin. Complete bitemporal hemianopia Retrochiasmal etiology Superior bitemporal hemianopia Pituitary adenoma, sellar meningioma Inferior bitemporal hemianopia Craniopharyngioma, hypothalamic tumor Central bitemporal hemianopic scotoma Hydrocephalus (third ventricle), craniopharyngioma, pinealoma Pseudobitemporal hemianopia Enlarged blind spot * Optic nerve etiology: (a) disc drusen, peripapillary atrophy, tilted nerves or (b) optic neuritis/chiasmitis * Retinal etiology: Retinitis pigmentosa, multiple evanescent white dot syndrome (MEWDS), acute zonal occult outer retinopathy (AZOOR) * High myopia Central scotoma/ centrocecal scotoma * Refractive error caused by improper lens correction * Toxic/metabolic optic neuropathies (ethambutol)

Case #23: Normal ESR... Could It Still Be Giant Cell Arteritis?


Jacinthe Rouleau MD
Answer and Teaching Points
I. Introduction A. Giant cell arteritis (GCA) is a systemic vasculitis that affects medium and large arteries. B. Accurate diagnosis and early treatment are important because GCA can lead to irreversible visual loss. II. History and Physical Examination A. Systemic manifestations 1. Headache 2. Scalp tenderness 3. Jaw/tongue claudication 4. Prominent/painful/pulseless temporal arteries 5. Weight loss/anorexia 6. Low-grade fever

Case #22: What to Do With This Bitemporal Hemianopia


Guy V Jirawuthiworavong MD
Answer
MRI of the brain was reanalyzed and revealed no acute hemorrhage or infarction. The parasellar region appeared normal, and there was no hydrocephalus. MRA of the head did not show an aneurysm. Fluorescein angiography was unremarkable. However, red free photos hinted to subtle petaloid lesions parafoveally and OCT showed abnormal thinning along the same areas O.U. OCT of the nerve fiber layer was unremarkable. Further analysis of the visual fields showed that they did not respect the vertical meridian (pseudobitemporal hemianopia). Acute macular neuroretinopathy O.U. (an idiopathic rare inflammatory condition) was diagnosed. Six weeks later, patients vision improved to 20/50 O.U. but her central visual defect persisted. One year later, no improvement was noted.

2011 Subspecialty Day

Neuro-Ophthalmology

Section IV: What Is Wrong With This Test?

39

7. Fatigue 8. Myalgias/arthralgias 9. Sore throat/hoarseness 10. Scalp/tongue necrosis 11. Stroke/transient ischemic attack 12. Angina/myocardial infarction 13. Mental status changes B. Ocular manifestations 1. Transient visual loss 2. Optic nerve a. Anterior ischemic optic neuropathy (AION) b. Posterior ischemic optic neuropathy (PION) 3. Retina a. Central retinal artery occlusion (CRAO) b. Cilioretinal artery occlusion c. Cotton wool spots 4. Choroidal ischemia 5. Diplopia a. Transient or fixed b. From ischemia of the extraocular muscles, cranial nerves, or brainstem 6. Ocular ischemic syndrome 7. Orbital pain 8. Pupil (rarely anisocoria as a presenting sign of GCA) a. Tonic pupil b. Horner syndrome 9. Visual field defects from cortical infarction 10. Visual hallucinations C. Features that raised the suspicion of GCA1 1. AION with: a. Poor visual acuity (eg, hand motions or worse vision) b. Chalk-white appearance of the optic nerve swelling (ie, pallid edema) c. Associated areas of retinal ischemia (cotton wool spot) or choroidal ischemia (visible on fluorescein angiogram) d. A large cup-to-disc ratio in the contralateral disc (ie, not the disc at risk for nonarteritic AION) e. History of amaurosis fugax or transient double vision preceding the AION

2. Posterior ischemic optic neuropathy (PION) in the absence of prior surgery, severe blood loss, or hypotension 3. Rapid sequential ophthalmic vascular events, especially if affecting separate vascular territories or affecting both eyes III. Laboratory Testing A. Erythrocyte sedimentation rate (ESR) 1. Most widely used laboratory test when GCA is clinically suspected 2. Reflects levels of many plasma proteins, responds slowly to inflammatory stimulus, and many factors can decrease its value (see Table 1). 3. The International Committee for Standardization in Hematology (ICSH) recommends the use of Westergren method for measurement of ESR. 4. Criterion used for determination of normal range can vary. a. American College of Rheumatology classification of GCA: ESR greater than 50 mm/h is 1 of the 5 criteria for diagnosis. b. Most commonly used formulas for upper limits of normal2 i. Men: Age divided by 2 ii. Women: Age plus 10, divided by 2 c. Other formulas proposed3 i. Men: 17.3 plus (0.18 X age) ii. Women: 22.1 plus (0.18 X age) 5. Normal ESR a. Does not exclude GCA b. Up to 17% of patients with GCA have normal ESR. c. If ESR is normal despite high clinical suspicion of GCA, search for factors that could decrease the ESR (Table 1), especially for current or recent treatment with any form of steroids (oral, injection, inhalers, sprays, creams). d. A subgroup of patients are unable to develop a full-blown acute-phase response and will have normal ESR. Tendency to have less frequent systemic findings also. e. Because ESR responds slowly to inflammatory stimulus, it can be low at the beginning of the disease and can become high if repeated in time. B. C-reactive protein (CRP) 1. Unaffected by age and gender 2. Reflects value of a single acute-phase protein produced by hepatocytes and has a rapid response to inflammatory stimulus

40

Section IV: What Is Wrong With This Test?

2011 Subspecialty Day

Neuro-Ophthalmology

Table 1. Factors That May Decrease ESR Measurements Technical Factors


Low room temperature Inadequate mixing Dilutional problem Specimen too old Vibration during testing Short tube

Pathological Factors
Polycythemia Extreme leukocytosis Sickle cell anemia Dysproteinemia with hyperviscosity state Hypofibrinogenemia Hypogammaglobulinemia Dysfibrinogenemia and afibrinogenemia Spherocytosis Microcytosis Acanthocytosis Anisocytosis Cachexia Disseminated intravascular coagulation

Medication
Anti-inflammatory agents Salicylates Nonsteroidal anti-inflammatory drugs Corticosteroids Statins Valproic acid Low molecular weight dextran

3. More sensitive for GCA compared to ESR 4. More expensive than ESR 5. ESR and CRP combined: Diagnosis sensitivity for GCA increases to 99%. C. Thrombocytosis ( > 400 00/L) Recent studies suggest that CRP and thrombocytosis may be stronger predictors of positive biopsy than ESR.4 1. Odds of a positive temporal artery biopsy: a. 1.5 times greater with ESR > 47mm/hr b. 5.3 times greater with a CRP > 2.45 mg/dL c. 4.2 times greater with platelets > 400 000/L D. Normocytic normochromic anemia 1. Present in 20%-50% of GCA cases 2. Less useful for diagnosis of GCA because the specificity of anemia in the elderly is very poor E. Other tests (not generally recommended) can be elevated: thrombopoietin, interleukin-6, von Willebrand factor, haptoglobin, orosomucoid, 1-antitrypsin and complement. IV. Conclusion A. Detailed history and physical examination are essential to determine the clinical likelihood of GCA. B. When there is a clinical suspicion for GCA, immediate laboratory investigation should be done, but its important to determine whether there is a high clinical suspicion for GCA regardless of the laboratory values (ie, pretest likelihood of disease). C. A normal ESR does not rule out GCA.

D. Possible reasons for normal ESR in GCA: 1. Presence of factor decreasing the ESR (eg, technical, pathological, or drug-related) (Table 1) 2. Subgroup of patients unable to develop a fullblown acute-phase response 3. Criterion used for determination of normal range 4. Short duration of symptoms (ESR not elevated yet) E. CRP and platelets count should be obtained in addition to ESR. F. High-dose corticosteroid treatment should be started immediately and a temporal artery biopsy should be obtained if laboratory inflammatory markers are abnormal or if the clinical suspicion for GCA remains high despite normal blood work.

References
1. Melson MR, Weyand CM, Newman NJ, Biousse V. The diagnosis of giant cell arteritis. Rev Neurol Dis. 2007; 4:128-142. 2. Miller A, Green M, Robinson D. Simple rule for calculating normal erythrocyte sedimentation rate. Br Med J (Clin Res Ed). 1983; 286:266. 3. Hayreh SS, Podhajsky PA, Raman R, et al. Giant cell arteritis: validity and reliability of various diagnostic criteria. Am J Ophthalmol. 1997; 123:285-296. 4. Walvick MD, Walvick MP. Giant cell arteritis: laboratory predictors of a positive temporal artery biopsy. Ophthalmology. Epub ahead of print 2011 Jan 11. 5. Kale N, Eggenberger E. Diagnosis and management of giant cell arteritis: a review. Curr Opin Ophthalmol. 2010; 21:417-422. 6. Jou JM, Lewis S M, Briggs C, Lee S-H, De la Salle B, McFadden S, for the International Council for Standardization in Haematology (ICSH). ICSH review of the measurement of the erythocyte sedimentation rate. Int J Lab Hematol. 2011; 33:125-132.

2011 Subspecialty Day

Neuro-Ophthalmology

Section IV: What Is Wrong With This Test?

41

7. Lawrence JS. Sources of error in the erythrocyte sedimentation rate. Ann Rheum Dis. 1953; 12:206-211. 8. Brigden ML. Clinical utility of the erythrocyte sedimentation rate. Am Fam Physician. 1999; 60:1443-1450. 9. Lane SK, Gravel JW Jr. Clinical utility of common serum rheumatologic tests. Am Fam Physician. 2002; 65:1073-1081. 10. Jurado RL. Why shouldnt we determine the erythrocyte sedimentation rate? Clin Infect Dis. 2001; 33:548-549. 11. Saadeh C. The erythrocyte sedimentation rate: old and new clinical applications. South Med J. 1998; 91:220-225. 12. Brigden M. The erythrocyte sedimentation rate: still a helpful test when used judiciously. Postgrad Med. 1998; 103:257-262, 272274. 13. Sox HC Jr, Liang MH. The erythrocyte sedimentation rate: guidelines for rational use. Ann Intern Med. 1986; 104:515-523. 14. Man PY, Dayan MR. Giant cell arteritis with normal inflammatory markers. Acta Ophthalmol Scand. 2007; 85:460. 15. Danesh-Meyer HV, Savino PJ. Giant cell arteritis. Curr Opin Ophthalmol. 2007; 18:443-449. 16. Salvarani C, Cantini F, Boiardi L, Hunder GG. Laboratory investigations useful in giant cell arteritis and Takayasus arteritis. Clin Exp Rheumatol. 2003; 21:S23-28. 17. Tehrani R, Ostrowski RA, Hariman R, Jay WM. Giant cell arteritis. Semin Ophthalmol. 2008; 23:99-110. 18. Hall JK. Giant-cell arteritis. Curr Opin Ophthalmol. 2008; 19:454460. 19. Nusser JA, Howard E, Wright D. Clinical inquiries: which clinical features and lab findings increase the likelihood of temporal arteritis? J Fam Pract. 2008; 57:119-120. 20. Niederkohr RD, Levin LA. Management of the patient with suspected temporal arteritis: a decision-analytic approach. Ophthalmology 2005; 112:744-756. 21. Smetana GW, Shmerling RH. Does this patient have temporal arteritis? JAMA 2002; 287:92-101. 22. Lawrence JS. Sources of error in the erythrocyte sedimentation rate. Ann Rheum Dis. 1953; 12:206-211. 23. Wise CM, Agudelo CA, Chmelewski WL, McKnight KM. Temporal arteritis with low erythrocyte sedimentation rate: a review of five cases. Arthritis Rheum. 1991; 34:1571-1574. 24. Hegg R, Lee AG, Tagg NT, Zimmerman MB. Statin or nonsteroidal anti-inflammatory drug use is associated with lower erythrocyte sedimentation rate in patients with giant cell arteritis. J Neuroophthalmol. 2011; 31:135-138.

Case #24: Thin Retinal Nerve Fiber Layer... Really??


Timothy James McCulley MD
Answer and Teaching Points
The accuracy of retinal nerve fiber layer (RNFL) thickness estimation is affected by the quality of the OCT images. Of course, many neuro-ophthalmic diseases can result in thinning of the RNFL. Distinguishing real disease from artifact or low/poor quality OCT images can be challenging. This is especially true in patients with both poor OCT signal strength and an optic neuropathy; determining the contribution of each may not be possible. Fortunately, many cases are straightforward. Simply looking at the entire exam can raise ones suspicion. In the case presented, the lack of a relative afferent pupillary defect, the lack of a RNFL type defect on visual field testing, and the normal fundus appearance strongly point to the possibility of a poor quality OCT. A low signal can be due to any type of media opacity: cataract, corneal scar, posterior capsular opacity, or vitreous hemorrhage. Other causes include a small pupil, poor fixation, or even a high refractive error. Some machines correct for refractive error. Some more modern machines actually record and average together many B-scan images to create a higher-resolution, higher-signal image from lower-signal images. Unfortunately, many OCT systems dont have the ability to track fundus features, and without fundus tracking a low signal can be hard to overcome. Numerous studies have looked at the influence of cataracts on OCT quality and their effect on accuracy of RNFL thickness measurements.1-3 Because of this there have been a variety of proposed indices that can be used to assess the accuracy/quality of an OCT image.4 Although this can be helpful, often closely looking at the entire clinical picture can provide the most practical insight. In the case presented, poor OCT signal strength in the left eye relative to the right eye is the most likely explanation for the asymmetric appearance of the RNFL. The lack of supporting evidence for pathology beyond a cataract in the setting of asymmetric media opacity is sufficient to support attributing the visual loss to the left eye cataract. Unnecessary additional testing can be avoided.

References
1. Van Velthoven ME, van der Linden MH, de Smet MD, Faber DJ, Verbraak FD. Influence of cataract on optical coherence tomography image quality and retinal thickness. Br J Ophthalmol. 2006; 90(10):1259-1262; erratum: 2010; 94(12):1695. 2. Kim NR, Lee H, Lee ES, et al. Influence of cataract on time domain and spectral domain optical coherence tomography retinal nerve fiber layer measurements. J Glaucoma. Epub ahead of print 16 December 2010. 3. Samarawickrama C, Pai A, Huynh SC, Burlutsky G, Wong TY, Mitchell P. Influence of OCT signal strength on macular, optic nerve head, and retinal nerve fiber layer parameters. Invest Ophthalmol Vis Sci. 2010; 51(9):4471-4475. 4. Liu S, Paranjape AS, Elmaanaoui B, et al. Quality assessment for spectral domain optical coherence tomography (OCT) images. Proc SPIE. 2009; 7171:71710X.

42

Section IV: What Is Wrong With This Test?

2011 Subspecialty Day

Neuro-Ophthalmology

Case #25: Homonymous Hemianopia: Wheres the Lesion?


Sushma Yalamanchili MD
Answer and Teaching Points
Differential diagnosis Homonymous hemianopia: cerebral infarction, intracranial hemorrhage, brain tumors, head trauma, and surgical interventions Diagnosis Neuropsychologic testing revealed visuospatial deficits Positron emission tomography (PET) imaging revealed slower metabolism (hypoperfusion) in the parieto-temporo-occipital region on the left side compared with the right. Conclusion Visual symptoms might be the presenting (or, rarely, the only) manifestation of Alzheimer disease. Clinicians should be aware of the visual variant of Alzheimer disease. Neuropsychological testing and MRI provide supporting evidence for the diagnosis. PET scans might be helpful in selected cases, especially those with a normal MRI.

Figure 1. From Morgan RK et al. Ophthalmology 1985; 92: 1356-1363. Reprinted with permission from Ophthalmology (Rochester, MN), American Academy of Ophthalmology.

Case #26: Is a Visual Evoked Potential in Nonorganic Visual Loss Needed Here?
Paul H Phillips MD
Answer
Abnormal VER in a patient with nonorganic visual loss

Discussion
The visual evoked response is an electrical response to a repetitive visual stimulus.1 Visual stimuli such as a flash or checkerboard pattern reversal (black square goes to white and white square goes to black) are presented to the patient, and the resulting electrical signals are recorded from the scalp over the occipital cortex. These signals are averaged to produce a waveform. The major component of the VER waveform is a positive deflection occurring approximately 100 milliseconds after the visual stimulus, often referred to as P100.

The P100 wave response is primarily generated by pyramidal cells in layer IV of the primary visual cortex.2 The amplitude and latency of the P100 wave is commonly used to evaluate the visual pathway. This P100 response is dependent on stimulus characteristics and optical factors, as well as an intact visual pathway. Important stimulus characteristics include luminance, contrast, checkerboard pattern size, and the frequency of pattern reversal. Optical factors are required to produce a clear retinal image. Finally, the VER depends on an intact visual pathway from the retinal photoreceptors to the optic nerves, optic tracts, lateral geniculate nucleus, optic radiations, and finally the primary visual cortex. An abnormality anywhere along this pathway may affect the VERaltering the P100 latency, the amplitude, or both. Conversely, a VER with a normal P100 amplitude and latency often indicates normal visual function. Thus, in patients with suspected functional visual loss that do not cooperate with subjective visual tests such as visual acuity and fields, a normal VER may be used as objective confirmation of normal visual function.3,4 However, several investigators have shown that patients with normal vision are able to voluntarily alter or extinguish their pattern VER.5-8 Uren et al5 obtained normal VERs in 20 healthy subjects and then repeated VER testing while the subjects were instructed to consecutively perform saccadic eye movements, fixate on the periphery of the pattern, and voluntarily defocus the pattern. All of these maneuvers could alter or extinguish the VER, simulating responses obtained in patients with organic disease such as optic neuritis. Bumgartner et al6 reported that 5 of 15 normal adults could alter or abolish their pattern VER using techniques such as meditation, daydreaming, and convergence. These maneuvers were inapparent to an observant technician. All subjects specifically instructed to use convergence acquired the ability to alter their VER.

2011 Subspecialty Day

Neuro-Ophthalmology

Section IV: What Is Wrong With This Test?

43

Tan et al7 reported that 7 of 12 normal subjects could deliberately alter or abolish their VER either by accommodation or eccentric fixation. The presence of accommodation was not detectable with monocular testing. However, adduction from convergence was noted upon removal of occlusion of the nontested eye. The VER normalized with addition of a minus lens to counteract the myopia induced by accommodation. They noted that use of a large stimulus size and binocular testing reduce the ability of subjects to voluntarily alter their VER, but also reduce the sensitivity of the VER for the detection of organic disease. In the study by Morgan et al,8 8 of 42 subjects were able to alter or extinguish their pattern VER without any change in fixation when instructed to ignore the pattern stimulus. They were unable to volitionally alter the flash VER. However, flash VER is less sensitive for the detection of organic disease of the visual pathway.

References
1. Baker RS, Schmeisser ET, Epstein AD. Visual system electrodiagnosis in neurologic disease of childhood. Pediatr Neurol. 1995; 12:99110. 2. Ducati A, Fava E, Motti EDF. Neuronal generators of the visual evoked potentials: intracerebral recording in awake humans. EEG Clin Neurol. 1988; 71:89-99. 3. Gundogan FC, Sobaci G, Bayer A. Pattern visual evoked potentials in the assessment of visual acuity in malingering. Ophthalmology 2007; 114:2332-2337. 4. Xu S, Meyer D, Yoser S, et al. Pattern visual evoked potential in the diagnosis of functional visual loss. Ophthalmology 2001; 108:7681. 5. Uren SM, Stewart P, Crosby PA. Subject cooperation and the visual evoked response. Invest Ophthalmol Vis Sci. 1979; 18:648-652. 6. Bumgartner J, Epstein CM. Voluntary alteration of visual evoked potentials. Ann Neurol. 1982; 12:475-478. 7. Tan CT, Murray NMF, Sawyers D, Leonard TJK. Deliberate alteration of the visual evoked potential. J Neurol Neurosurg Psychiatr. 1984; 47:518-523. 8. Morgan RK, Nugent B, Harrison JM, OConnor PS. Voluntary alteration of pattern visual evoked responses. Ophthalmology 1985; 92:1356-1363. 9. Burde RM. Voluntary alteration of pattern visual evoked responses [discussion]. Ophthalmology 1985: 92:1362-1363.

Teaching Points
Patients with functional visual loss can volitionally alter or extinguish the pattern VER using techniques such as meditation, daydreaming, and convergence.6 These maneuvers may be inapparent to an observant technician. The history and clinical examination remain the gold standard for the diagnosis of functional visual loss.9

2011 Subspecialty Day

Neuro-Ophthalmology

45

Financial Disclosure

The Academys Board of Trustees has determined that a financial relationship should not restrict expert scientific, clinical, or nonclinical presentation or publication, provided that appropriate disclosure of such relationship is made. As an Accreditation Council for Continuing Medical Education (ACCME) accredited provider of CME, the Academy seeks to ensure balance, independence, objectivity, and scientific rigor in all individual or jointly sponsored CME activities. All contributors to Academy educational activities must disclose any and all financial relationships (defined below) to the Academy annually. The ACCME requires the Academy to disclose the following to participants prior to the activity: any known financial relationships a meeting presenter, author, contributor, or reviewer has reported with any manufacturers of commercial products or providers of commercial services within the past 12 months any meeting presenter, author, contributor, or reviewer (hereafter referred to as the Contributor) who report they have no known financial relationships to disclose For purposes of this disclosure, a known financial relationship is defined as any financial gain or expectancy of financial gain brought to the Contributor or the Contributors family, business partners, or employer by: direct or indirect commission; ownership of stock in the producing company; stock options and/or warrants in the producing company, even if they have not been exercised or they are not currently exercisable; financial support or funding from third parties, including research support from government agencies (e.g., NIH), device manufacturers, and/or pharmaceutical companies; or involvement in any for-profit corporation where the Contributor or the Contributors family is a director or recipient of a grant from said entity, including consultant fees, honoraria, and funded travel. The term family as used above shall mean a spouse, domestic partner, parent, child or spouse of a child, or a brother, sister, or spouse of a brother or sister, of the Contributor.

Category Consultant / Advisor

Code C

Description Consultant fee, paid advisory boards or fees for attending a meeting (for the past one year) Employed by a commercial entity Lecture fees (honoraria), travel fees or reimbursements when speaking at the invitation of a commercial entity (for the past one year) Equity ownership/stock options of publicly or privately traded firms (excluding mutual funds) with manufacturers of commercial ophthalmic products or commercial ophthalmic services Patents and/or royalties that might be viewed as creating a potential conflict of interest Grant support for the past one year (all sources) and all sources used for this project if this form is an update for a specific talk or manuscript with no time limitation

Employee Lecture fees

E L

Equity owner

Patents / Royalty

Grant support

46

2011 Subspecialty Day

Neuro-Ophthalmology

2011 Neuro-Ophthalmology Planning Group Financial Disclosures


Andrew G Lee MD
None

AAO Staff Financial Disclosures Ann LEstrange


None

Michael S Lee MD
Pfizer, Inc. S Quark Pharmaceuticals S Teva Pharmaceutical Industries S

Melanie Rafaty
None

Madhu R Agarwal MD
None

Debra Rosencrance
None

Eric L Berman MD
None

Wayne T Cornblath MD
None

Matthew Dean Kay MD


None

Leah Levi MBBS


None

Alfredo A Sadun MD PhD


Edison Pharmaceuticals S Pfizer Ophthalmics C

Prem S Subramanian MD PhD


Lundbeck, Inc. S Novartis Pharmaceuticals Corp. S Pfizer, Inc. S Teva Pharmaceutical Industries S

2011 Subspecialty Day

Neuro-Ophthalmology

47

Faculty Financial Disclosures

Marie D Acierno MD
None

Eric Eggenberger DO
Berlex, Inc. C,L Biogen, Inc. C,L,S Serono C,S Teva Pharmaceutical Industries C,L,S

Andrew G Lee MD
None

Madhu R Agarwal MD
None

Michael S Lee MD
Pfizer, Inc. S Quark Pharmaceutical S Teva Pharmaceutical Industries S

Anthony C Arnold MD
Pfizer, Inc. L

Julie Falardeau MD
None

Rudrani Banik MD
None

Rod Foroozan MD
None

Reid A Longmuir MD
None

Valerie Biousse MD
None

Larry P Frohman MD
None

Timothy J McCulley MD
None

Gabrielle R Bonhomme MD
None

Karl C Golnik MD
Alcon Laboratories C

Neil R Miller MD
National Eye Institute S Quark Pharmaceuticals C

Francois-Xavier Borruat MD
Allergan, Inc. L Novartis Pharmaceuticals C

Lynn K Gordon MD PhD


VentiRx Pharmaceuticals C

Mark L Moster MD
Biogen, Inc. L Teva Pharmaceutical Industries L

Beau Benjamin Bruce, MD MS


National Eye Institute S Pfizer, Inc. S Teva Pharmaceutical Industries S

Thomas A Graul MD
None

Nancy J Newman MD
Biogen, Inc. C Quark Pharmaceuticals C

Guy V Jirawuthiworavong MD
None

Dean M Cestari MD
None

Michael C Johnson MD
None

Steven A Newman MD
None

Sophia Mihe Chung MD


National Eye Institute S Pfizer, Inc. S

Randy H Kardon MD PhD


Department of Defense TATRC S National Eye Institute S Novartis Pharmaceuticals Corp. C Veterans Administration S

Jeffrey G Odel MD
Bayer Health Care C

Paul H Phillips
None

Kimberly Cockerham MD FACS


Acktivatek C Department of Defense S FanMinder C North American Neuro-Ophthalmology Society S

Matthew Dean Kay MD


None

Peter A Quiros MD
None

Lanning B Kline MD
None

Joseph F Rizzo III MD


None

Wayne T Cornblath MD
None

Gregory S Kosmorsky DO
None

Jacinthe Rouleau MD
Santhera Pharmaceuticals C

Fiona E Costello MD
Novartis Pharmaceuticals Corp. C

Byron L Lam MD
CDC S Department of Defense S NASA C National Eye Institute S Pfizer, Inc. S Quark S

R Michael Siatkowski MD
National Eye Institute S

Kathleen B Digre MD
None

Prem S Subramanian MD PhD


Lundbeck, Inc. S Novartis Pharmaceuticals Corp. S Pfizer, Inc. S Teva Pharmaceutical Industries S

48

Faculty Financial Disclosures

2011 Subspecialty Day

Neuro-Ophthalmology

Nathan Troy Tagg MD


None

Roger E Turbin MD
Lundbeck C,L Pfizer, Inc. S

Michael S Vaphiades DO
None

Nicholas J Volpe MD
None

Sushma Yalamanchili MD
None

2011 Subspecialty Day

Neuro-Ophthalmology

49

Presenter Index

Acierno, Marie D 7, 24 Banik, Rudrani 2, 16 Bonhomme, Gabrielle R 10, 31 Borruat*, Francois-Xavier 7, 23 Bruce*, Beau Benjamin 11, 33 Cestari, Dean M 5, 21 Cockerham*, Kimberly 11, 34 Costello*, Fiona E 3, 19 Digre, Kathleen B 8, 25 Falardeau, Julie 11, 33 Foroozan, Rod 10, 31 Graul, Thomas A 6 Jirawuthiworavong, Guy V 12, 38 Johnson, Michael C 2, 16 Kardon*, Randy H 12, 37 Lam*, Byron L 3, 18 Longmuir, Reid A 8, 27 McCulley, Timothy J 13, 41 Moster*, Mark L 10, 30 Phillips, Paul H 14, 42 Quiros, Peter A 11, 32 Rouleau*, Jacinthe 13, 38 Siatkowski*, R Michael 4, 19 Tagg, Nathan Troy 9, 28 Turbin*, Roger E 4, 20 Vaphiades, Michael S 8, 26 Yalamanchili, Sushma 13, 42

* Indicates that the presenter has financial interest. No asterisk indicates that the presenter has no financial interest.

También podría gustarte